Download AdultHealthAnswerKey

Document related concepts
no text concepts found
Transcript
CHAPTER 1—INTRODUCTION TO ANATOMY AND PHYSIOLOGY
Crossword Puzzle
Figure Labeling: Planes of the Body
2.
The sagittal plane runs lengthwise from the front to the back. A sagittal cut gives a right and a left portion of the
body. A midsagittal cut gives two equal halves. The coronal (frontal) plane divides the body into a ventral (front)
section and a dorsal (back) section. The transverse plane cuts the body horizontal to the sagittal and frontal planes,
dividing the body into caudal and cranial portions. See Figure 1-2 for additional information.
Table Activity
3.
See Table 1-6 for additional information.
One Body Part of
Major System
Major System
Function
Lungs
Respiratory
Exchange of carbon dioxide for oxygen; regulation of acid-base
balance
Blood vessels
Cardiovascular
Transportation of nutrition, water, oxygen, and wastes
Brain
Nervous
Coordination of body’s activities; communication
Stomach
Digestive
Mechanical and chemical breakdown of food; absorption of
nutrients
Kidneys
Urinary
Clearing blood of waste products; water and electrolyte balance;
acid-base balance
Bones
Skeletal
Support; movement; storage of minerals; blood cell formation
Voluntary muscles
Muscular
Movement; maintenance of posture; heat production
Skin
Integumentary
Protection; regulation of body temperature; synthesis of
chemicals; sense organ
Thyroid gland
Endocrine
Production of hormones that affect metabolism
Lymph nodes
Lymphatic
Protection
Gonads
Reproductive
Production of sex cells
Multiple Choice
4. Answer 4: Nursing documentation should include medical terminology and specific details that describe assessment
findings, interventions, or evaluation. {Note to student: patient statements can be used in nursing documentation
but should be placed in quotation marks.]
5. Answer 2: Nursing documentation is a legal record of care that was given to the patient. This note shows the patient
had a position change every two hours. [Note to student: Based on the type of risks that the patient has, the nurse
might also use additional interventions every 2 hours, for example: skin assessment, range-of motion, incentive
spirometer, toileting, or oral fluids].
6. Answer 3: Dorsal or posterior is towards the back of the body. The perineal area is inferior to the umbilicus. The front
of the chest is the anterior surface. The face and top of the head are the superior to the rest of the body.
7. Answer 1: The dorsalis pedis pulse is on the top of the foot and it is the most distal pulse in the lower extremity. The
popliteal is behind the knee (can be difficult to locate). The posterior tibial pulse is behind the medial malleolus
(lateral ankle). The femoral pulse is in the groin area.
8. Answer 3: Medial is towards the midline. The sternum is in the middle of the chest.
9. Answer 4: The plasma membrane contains surface proteins that identify a cell as coming from one particular
individual. Tissue typing is performed for compatibility to reduce the chances of rejection of a transplanted organ
10. Answer 1: A muscle cell cannot operate properly unless the intracellular Ca2+ concentration is kept low during rest.
11. Answer 2: Hematopoietic disorders affect the blood cell–producing cells located in the red bone marrow; a decreased
red blood count is anticipated. Potassium and glucose are electrolytes. Blood urea nitrogen reflects kidney
function.
12. Answer 3: The spleen plays important role in the immune function. Loss of the spleen increases the patient's risk for
developing infections.
13. Answer 3: The gallbladder is located just below the right ribs. The spleen is on the left side. The small intestine and
cecum are located in the lower abdominopelvic cavity.
14. Answer 2: The urinary bladder is located in the hypogastric region. See Figure 1-4 for additional information.
15. Answer 1: The stomach is located in the epigastric region. See Figure 1-4 for additional information.
16. Answer 2: The appendix is located in the right lower quadrant. See Figure 1-5 for additional information.
17. Answer 4: Once diagnosed, patients are usually placed on “nothing by mouth” (NPO), but a patient who develops a
small bowel obstruction at home will often seek health care because of vomiting and abdominal pain. A proximal
obstruction is one that is closer to the beginning of the small intestine; therefore, the blockage is higher up in the
system. Vomiting can occur whenever there is an intestinal obstruction; however, in a distal large intestinal
obstruction, vomiting is less likely. If it develops, it usually occurs later and the emesis could have a fecal odor.
18. Answer 1: The epidermis or skin is composed of stratified squamous tissue. One of the main functions is to protect
the body from infection. Bones are for strength and structure. Simple columnar tissue participates in the secretion of
mucus. Adipose tissue provides insulation.
19.
20.
Answer 3: The mucous membranes are designed to trap microorganisms and dryness decreases that function. Poor
oral hygiene contributes to respiratory infection, especially for patients who are bedridden. Patients who are in a
coma are not given solid food. Dignity and preservation of the teeth are desirable for all patients.
Answer 3: The bursae are small cushionlike sacs that are found between joints; therefore, the nurse would assess the
movement and discomfort of the major joints.
Critical Thinking Activities
Activity 1
21. Knowledge of how the body works helps the nurse to distinguish normal findings from abnormal findings.
Knowledge of location and function of organs helps the nurse predict the involvement of underlying structures that
are related to patients’ reports of pain and discomfort and design interventions that will enhance function or repair
dysfunction. Knowledge of physiology at the cellular level helps the nurse implement interventions that keep the
body in homeostasis.
Activity 2
22. 2-cm ecchymosis noted on distal tip of first digit of right foot.
Activity 3
23. Accuracy is an important part of documentation; thus using the patient’s words in direct quotes is acceptable. In
addition, assessment data should reflect the nurse’s ability to make and record professional observations. When the
nurse’s records are reviewed by other health care professionals or by legal or financial consultants, use of correct
terminology and accuracy reflect the quality of care.
CHAPTER 2—CARE OF THE SURGICAL PATIENT
Table Activity
1.
Assessment
Normal Findings
Frequency
a.
Vital signs
Same as or close to preoperative
q 15 minutes x 4, q 30 minutes x 4, q 60
minutes x 4, q 4 hours x 4, until
assessments are within normal range
b.
Incision
Dressing dry and intact; no drainage
Every time vital signs are assessed
c.
Respiratory
effort
Breathing comfortably without
dyspnea
q 1-2 hours
d.
Pain
Relieved by analgesics and
nonpharmacologic methods
Assess concurrently with vital signs
e.
Urinary function
Voids at least 30 mL/hour
Within 6-8 hours of surgery
f.
Neurovascular
Extremities are warm, pulse present;
skin color normal for patient
q 2 hours
g.
Activity
Muscle-strengthening exercises,
sitting, dangling, and walking;
gradual progression towards baseline
Per health care provider's orders and
patient’s ability
h.
Gastrointestinal
function
Flat abdominal area; bowel sounds
audible in four quadrants
q 2 hours
Matching
2.
d
3.
a
4.
f
5.
e
6.
b
7.
g
8.
c
9.
i
10. h
Multiple Choice
11. Answer 2: Patients are generally encouraged to eat well-balanced diets, but protein foods are specific for building
and repairing body tissues.
12. Answer 1: Phenytoin (Dilantin) is a antiseizure medication. Antiseizure and cardiac medications are sometimes given
with a sip of water. The surgeon or anesthesiologist should be contacted if there is a query. Warfarin sodium
(Coumadin), an anticoagulant, is usually discontinued several days before the surgery. Ranitidine (Zantac) is an
antiulcer medication and may be given prophylactically in the post-surgical period. Acetaminophen (Tylenol) is
used postoperatively for mild pain.
13. Answer 1, 2, 3, 5: Surgeon provides information regarding the actual surgical procedure, as well as the risks, benefits,
and possible outcomes. Nurse is responsible for teaching about the other topics.
14. Answer 4: Abnormal potassium levels can cause cardiac dysrhythmias. All of the values are slightly above normal
and should be reported. Normal ranges include: potassium 3.5 to 5.0 mEq/L; sodium 135 to 145 mEq/L; glucose 70
and 100 mg/dL; and blood urea nitrogen 10 to 20 mg/dL
15. Answer 3: Urinary catheters are frequently made of latex and swelling is one symptom of allergic response. Throat
discomfort after nasogastric tube insertion is not uncommon. Hematoma after a venipuncture could be related to
poor technique or failure to provide pressure at the site; however, the nurse would follow up to see if the patient is
on any medication (e.g., warfarin) or had health problems (e.g., hemophilia) that would interfere with clotting.
Dermabrasion is a surgical procedure of scraping away skin.
16. Answer 2: Complete physical assessment and history are always important; however, the assessments that are most
relevant to the skin preparation, relate to skin impairment (e.g., infection, irritation, bruises, lesions) and skin
allergies. Skin infections could be a source of wound infection. Bruises, irritation and lesions should be noted as preexisting conditions of the surgical procedure. Skin allergies impact the type of solutions, medications and
equipment that contacts the skin. Any abnormal assessment findings are recorded and reported to the surgeon.
17. Answer 4: An oral airway can be inserted to keep the airway open (keeps tongue from obstructing airway). Bagvalve-mask is used when patients are not breathing. Oral suctioning (also, side lying position or elevation of the
head) decreases the likelihood of aspirating secretions. Oxygen is administered to maintain saturation, but the
airway must be open for oxygen to pass into the lungs.
18. Answer 3: The nurse who does the preoperative care signs the list thereby taking responsibility for the care on the
list. Medication, tasks completed by delegation and teaching are included in the care prior to surgery.
19. Answer 2: Standard practice in most facilities is to monitor vital signs and make general assessments using the “times
4” factor—every 15 minutes times 4 (for 4 times); every 30 minutes times 4; every hour times 4; then every 4 hours,
until assessments are within expected ranges.
20. Answer 4: Post-operative patients are at risk for hypovolemic shock. Signs/symptoms include: hypotension,
tachycardia, restlessness, apprehension, and cold, moist, pale, or cyanotic skin. Standard protocol in most facilities
includes: (1) administer oxygen (2) raise legs above the level of the heart; (3) increase the rate of IV fluid; (4) notify
anesthesiologist and surgeon; (5) medications as ordered; and (6) assess response to interventions.
21. Answer 4: In the induction phase, the patient is awake and the administration of anesthetic agents begins. The stage is
completed when the patient loses consciousness, and endotracheal intubation is established and placement verified.
22. Answer 2: Anesthesia may be maintained through a combination of inhalation and IV medications. Emergence from
anesthesia occurs when the procedure is completed and reversal agents are given.
23.
24.
25.
26.
27.
28.
29.
30.
31.
32.
33.
34.
35.
36.
37.
38.
39.
40.
41.
Answer 3: Spinal anesthesia is often used for lower abdominal, pelvic, and lower extremity procedures; urologic
procedures; or surgical obstetrics.
Answer 2: Local anesthesia is commonly used for minor surgical procedures, such as a biopsy of a superficial skin
lesion.
Answer 4: Combinations of sedatives, tranquilizers, anesthetics, or anesthetic gases are commonly used for
moderate sedation. The health care provider is frequently focused on the procedure and relies on the nurse to
monitor the patient. Monitoring vital signs is necessary to detect adverse effects of the medication or the
procedure.
Answer 3: Resuscitation equipment must be readily available in case the patient has respiratory depression or
cardiac dysrhythmia. Recovery is rapid and relatively less risky than other types of anesthesia. The patient is not
routinely intubated.
Answer 3: For Arab Americans, verbal consent often has more meaning than written consent because it is based on
trust. The need for written consent should be fully explained.
Answer 1: Teaching 1 or 2 days before surgery is ideal because the patient’s anxiety is not too high. Teaching too far
in advance would affect retention of the information. The teaching should not be delayed (or hastened) because of
the nurse’s schedule.
Answer 3: Before bowel surgery, medication (neomycin, sulfonamides, erythromycin) may be given over a period of
days to detoxify and sterilize the gastrointestinal tract.
Answer 2, 3, 4: Antihypertensives interact with anesthetic agents to cause bradycardia, hypotension, and impaired
circulation.
Answer 1: Nonsteroidal antiinflammatory drugs inhibit platelet aggregation and may prolong bleeding, increasing
susceptibility to postoperative bleeding.
Answer 2: In the immediate postoperative period, all patients are at risk for aspiration related to nausea and
vomiting and will have impaired abilities to manage secretions. Elderly patients have additional problems related to
age.
Answer 2, 4, 6: The unlicensed assistive personnel (UAP) can assist the patient to remove any personal clothing and
don hospital attire and can also apply the antiembolic stockings. The UAP can assist the patient to move from the
bed to the stretcher. Comparing data, checking IV sites and equipment, and ensuring that the postoperative list is
completed are nursing responsibilities.
Answer 4: The patient may be feeling fear of the unknown or fear of cancer; long-term, she may be thinking about
death, mutilation, or change of lifestyle. First address the feelings and then ask her to expand on her fears. Based on
assessment findings, the other options might be used.
Answer 2: While all of these patients have the potential for adverse reactions and drug-drug interactions, the elderly
patient with polypharmacy and chronic health conditions is the most vulnerable.
Answer 2: Smoking increases the risk for pneumonia and atelectasis; use of the incentive spirometer decreases the
risk for respiratory complications. The patient’s reading on pulse oximeter is likely to be lower than normal or lownormal because of the smoking. Patient-controlled analgesia pump and call bell are also important, but less related
to the issue of smoking.
Answer 4: “What...?” is an open-ended question. This allows the patient to seek information and the nurse can
determine areas where the patient needs clarification. The other questions are closed-ended and do less to encourage
the patient to speak.
Answer 4: If consent is obtained while the patient is under the influence of consciousness-altering substances (even
if prescribed), the consent is not considered valid. The other information is also relevant, and the surgeon should be
advised.
Answer 2: The unlicensed assistive personnel (UAP) can assist with oral care; however, the patient and the UAP
should be instructed that fluids should not be swallowed. During nothing by mouth (NPO) status, patients usually
are not given any fluid. The exception could be small sips of water to take certain medications. Some surgeons will
allow the patient to have small hard candies, but sucking hard candies does stimulate peristalsis, so this is not
standard practice for all patients who are NPO. UAPs are not responsible for checking IV fluids.
Answer 3: Coughing increases intracranial pressure; therefore, coughing is contraindicated for patients with
intracranial surgery.
Answer 2: The nurse would check for distention first and then consider the other options.
42.
43.
44.
45.
46.
47.
48.
49.
50.
Answer 3: Slowing of the respiratory rate suggests that the level of anesthesia is causing respiratory paralysis; the
patient may require resuscitation. A decrease in blood pressure is also serious because of possible vasodilation. Loss
of sensation and decreased movement of the lower extremities are expected.
Answer 3: The nurse would assess the extremity for the new report of discomfort. Based on assessment findings, the
nurse could consider the other options. (Postoperatively, the patient could have an emboli or a deep vein thrombus.
Positioning on the operating table could put pressure on tissues or nerves. Patient could also have a problem that is
not directly related to surgery; for example, bursitis.)
Answer 1: The patient is instructed to get up and void before getting the medication because it causes most people
to get drowsy. Urinary retention is also a common complication after surgery. The surgeon should mark the site and
obtain consent. Most preoperative checklists require noting that the site has been marked and that the consent form
is signed. All laboratory tests are usually completed at this point.
Answer 1, 2, 4, 6: The unlicensed assistive personnel (UAP) can obtain most of the equipment but is not responsible
for checking the function of pumps or suction equipment. The nurse should ensure that these items are functional,
as they are likely to be needed when the patient arrives
Answer 4: The patient must have stable vital signs before he/she is transferred to the nursing unit. If the order for
transfer has been written, the postanesthesia care unit nurse would be responsible for informing the anesthesiologist
about the unstable vital signs. Nausea, vomiting, a sore throat, and wound pain are expected.
Answer 1: First the nurse would check the patient. If there are no obvious signs or symptoms of shock, then the
nurse would instruct the unlicensed assistive personnel (UAP) to take and report BP and pulse to determine a trend.
A lower-than-baseline blood pressure is not uncommon after surgery.
Answer 4: The scrub nurse performs actions that require sterile handling. The circulating nurse is considered
nonsterile and can perform tasks that require asepsis. He/she helps the scrub nurse and surgeons maintain sterility.
Answer 3: The ambulatory surgery patient is released to home, so the patient must be alert and pain, nausea, and
vomiting must be controlled. The patient is not allowed to drive himself home and family’s willingness to assume
responsibility does not absolve the nurse from making decisions about the patient’s safety.
Answer 3: Any of these findings warrant further investigation; however, for diabetic patients, there is an increased
susceptibility for infection and poor wound healing. Impaired communication can be a problem for patients who
have had a cerebrovascular accident. Bloody emesis could be related to esophageal varices. Hypoventilation is a
problem for patients with preexisting respiratory disorders.
Critical Thinking Activities
Activity 1
51 Latex allergy occurs in three ways: (1) as irritant contact dermatitis, (2) as type IV allergic reactions, and (3) as type I
allergic reactions. The irritant reaction, which is most commonly seen, is a nonallergic reaction and results in itchy,
dry, and irritated hands. The type IV allergic reaction to latex is a cell-mediated response to the chemical irritants
found in latex products. The true latex allergy is the type I allergic reaction, and it occurs shortly after exposure to
the proteins in latex rubber. The type I reaction is an immunoglobulin E–mediated systemic reaction that occurs
when latex proteins are touched, inhaled, or ingested
Factors influencing: Person's susceptibility and the route, duration, and frequency of latex exposure
Risk factors: History of anaphylactic reaction of unknown cause during a medical or surgical procedure,
multiple surgical procedures, food allergies (specifically kiwi, bananas, avocados, chestnuts), a job with daily
exposure to latex, history of reactions to latex; history of allergies and asthma
Methods of prevention: Screen prior to admission, provide a latex-free environment, communication to all
members of the health care team, clearly marking the chart
Activity 2
52. See Box 2-3.
Activity 3
53. Older patients have higher morbidity and mortality rates than younger patients.
Older individuals often have chronic conditions and polypharmacy that increases the risk for complications.
Recovery can be affected by the level of mental functioning, individual coping abilities, and the availability of
support systems. These are often altered in the older adult.
Risks of aspiration, atelectasis, pneumonia, thrombus formation, infection, and altered tissue perfusion are
increased in the older adult.
Disorientation or toxic reactions can occur in the older adult after the administration of anesthetics, sedatives, or
analgesics. Older adults often have a slower metabolism of these substances. These reactions may linger days after
administration.
CHAPTER 3—CARE OF THE PATIENT WITH AN INTEGUMENTARY DISORDER
Short Answer
1.
Protection from infection
Regulation of temperature
Synthesis of vitamin D
Prevention of dehydration
Excretion of waste
2.
P—Provocative and palliative factors (things that bring the condition on)
Q—Quality/quantity (characteristics and size) of the skin problem
R—Region (specific region of the body)
S—Severity (of the signs and symptoms)
T—Time (length of time the patient has had the disorder)
3.
A—Is the mole Asymmetrical?
B—Are the Borders irregular?
C—Is the Color uneven or irregular?
D—Has the Diameter of the growth changed recently?
E—Has the surface area become Elevated?
4.
Oxygenation, pulmonary function, cardiac function, blood count, temperature and elimination of waste byproducts.
5.
Assess for pallor by looking at the mucous membranes, lips, nail beds, conjunctivae of lower eyelids
6.
Palpation for warmth and induration
Figure Labeling—Rule of Nines
7.
See Figure 3-19.
8.
a. 36%
b. 63%
c. 18%
Multiple Choice
9. Answer 1: Anemia, (low hemoglobin and hematocrit), causes the skin to appear pale and unhealthy. Yellow
discoloration (jaundice) is usually associated with elevated levels of bilirubin (e.g., liver or gallbladder disorders).
Cyanosis is caused by decreased oxygenation to the tissues (e.g., chronic obstructive pulmonary disorder). Brown
spots related to concentrations of melanin are associated with aging.
10. Answer 4: A stage 1 pressure injury is a localized intact area with nonblanchable redness; typically, over a bony
prominence. The wound characteristics vary: areas may be painful, firm, soft, warm, or cool compared with adjacent
tissue. A stage 2 pressure injury is a partial-thickness loss of dermis. It appears as a shallow open injury, usually
shiny or dry, with a red-pink wound bed without slough or bruising. A stage 3 pressure injury involves fullthickness tissue loss; subcutaneous fat is sometimes visible. Possible features are undermining and tunneling. A
stage 4 pressure injury involves full-thickness tissue loss with exposed bone, tendon, cartilage, or muscle. Slough or
eschar may be present on the wound bed. Undermining or tunneling are frequently present.
11. Answer 4: Transmission rate of the herpes virus from an infected mother to a newborn is approximately 30%–50%
among women who acquire genital herpes near the time of delivery.
12. Answer 2: Antiviral drugs, such as acyclovir (Zovirax) can shorten the outbreak and lessen the severity of
13.
14.
15.
16.
17.
18.
19.
20.
21.
22.
23.
24.
25.
26.
27.
28.
29.
30.
31.
symptoms.
Answer 2, 3, 5: Persons who are immunocompromised have a greater risk of developing serious complications,
including death. Healthy adults usually do not have long-term sequalae from herpes zoster infections.
Answer 1: Cellulitis is an infection of the skin and underlying subcutaneous tissues. The affected areas become
erythematous, edematous, tender, and warm to the touch. These signs/symptoms can also occur with venous
thrombosis. Pityriasis rosea begins as a single scaly area up to 4 inches in diameter (10 cm) with a raised border and
a pink center that resembles ringworm (a fungal infection). Spider angioma is associated with liver disease. An
angioma develops when a group of blood vessels dilate and form a tumor-like mass. Tinea corporis produces flat
lesions that are clear in the center with erythematous borders. Scaliness also occurs, and pruritus is severe
Answer 3: Infected or inflamed hair follicles occur in folliculitis, furuncles, and carbuncles.
Answer 1: Common causes of dermatitis are detergents, soaps, industrial chemicals, and plants such as poison ivy.
The other questions would apply to different skin conditions.
Answer 4: Wheals (hives) signal an allergic reaction. Respiratory rate, and effort should be monitored.
Answer 3: Lindane (Kwell) is a treatment for lice. The environment must be treated to prevent reinfection. Bleeding
or oozing or a change in a lesion or mole is one of the warning signs of skin cancer that should be reported to the
health care provider. Use of sunscreen is recommend for protection against skin cancer and for patients that have
skin sensitivity (e.g., systemic lupus erythematosus). Patients with acne are advised to use neutral soaps; wash hair
and body to remove excess oil and excretions and to prevent odor; and to avoid hot water and vigorous rubbing.
Answer 2: Systemic lupus erythematosus is an autoimmune disorder. It is a chronic, multisystem inflammatory
disorder of exacerbations and remissions. Signs/symptoms of exacerbation include: fever, rash, cough, or increasing
muscle and joint pain. Eruption of vesicles preceded by pain is characteristic of herpes zoster. Erythema, pain, and
tenderness over an area of skin are the first signs/symptoms of cellulitis. A vesicle that appears, ulcerates, ruptures
and encrusts is characteristic of type 1 herpes simplex (cold sore).
Answer 2: Alopecia is hair loss, which is a common side effect of chemotherapy. Use of scarves or wigs could help.
Also teach the patient that the hair will grow back. Therapeutic baths and applying lotions after bathing help with
pruritus. Shaving, tweezing, or pumice stones can be used for hirsutism (excessive growth of hair in a masculine
distribution).
Answer 1: Paronychia is an infection of the nail that spreads around the nail. Topical antibiotics and wet dressings
are the usual treatment; sometimes a surgical incision and drainage of the infected area are performed.
Answer 4: Skin disease, endocrine problems, and malnutrition are associated factors for hypotrichosis.
Answer 3: The most likely diagnosis is cellulitis. The extremity should be immobilized and elevated and warm,
moist dressings are applied to relieve discomfort. Therapeutic baths are usually used for dry or itchy skin.
Answer 3: Eczema is associated with allergies to chocolate, wheat, eggs, and orange juice.
Answer 1: Isotretinoin (Accutane) is teratogenic; thus pregnancy is an absolute contraindication and strict
contraception is advised for 1 month before starting and 1 month after completing treatment. Avoiding sun
exposure is also advised.
Answer 2: A raised, black nevus is considered one of the most threatening skin lesions, and removal is
recommended to prevent it from becoming malignant. Any change in color, size, or texture or any bleeding or
pruritus deserves investigation. The other comments reflect typical changes associated with aging.
Answer 1: Clubbing of the fingertips indicates chronic hypoxemia, which is associated with conditions such as
emphysema.
Answer 2: The palm of the hand supplies more information about temperature and texture than the fingertips, and
both sides should be compared. A cotton-tipped applicator can be used to test for sensation. Use of gloves is
recommended if the skin is broken or if mucous membranes are being assessed.
Answer 3: The nurse may suspect self-mutilation, but must conduct further assessment. Based on the assessment,
the nurse might consider using the other options.
Answer 4: The eschar provides protection, so at the this point it is left intact. The RN and LPN/LVN would
collaborate to develop a comprehensive, long-term care plan, which may include the wound care specialist. The
pressure injury is currently unstageable because it can’t be fully assessed.
Answer 3: Health care staff who have received two doses of the varicella vaccine should be assessed for symptoms
8-21 days after exposure to the patient with shingles. Staff who develop symptoms consistent with herpes zoster
should be removed from active duty. Health care staff who have not received the two doses of varicella vaccine may
32.
33.
be infective for 8-21 days and should be moved to another duty location away from patient care.
Answer 3: Dermatitis medicamentosa can cause patients to have respiratory distress. Dermatitis venenata is caused
by contact with plants and the area should be immediately washed. Pain, itching, and infection are possible
complications for many skin disorders, but these problems have lower priority than respiratory distress.
Answer 4: Wheals and hives after exposure to foods, insect bites, drugs, and other allergens can lead to anaphylactic
shock. Epinephrine would be given to this patient for respiratory symptoms or if rapid worsening occurs. Herpes
simplex is accompanied by burning sensation and a dry, crusty lesions. Single pink, scaly patch that resembles a
large ringworm occurs with pityriasis rosea. Skin maceration, fissures, and vesicles around the toes is typical of tinea
pedis.
Critical Thinking Activities
Activity 1
34. a. Emergent phase: Stop the burning by removing clothes and shoes. Open the airway, control bleeding, and
remove all nonadherent clothing and jewelry. Cover the victim with clean sheet or cloth, assess ABCs (airway,
breathing, and circulation) and look for life-threatening injuries. Assess every 30 minutes to 1 hour. Initiate fluid
therapy, insert indwelling catheter, monitor intake and output every hour, insert nasogastric tube to prevent
aspiration, and administer analgesics in small, frequent doses.
b. Acute phase: ABCs—assess respiratory pattern, vital signs, circulation, intake and output, ambulation, bowel
sounds, wounds, and mental status. Control of pain decreases anxiety. Initiate protective measures for skin by
maintaining protective isolation. Dressing and treatment of burns as ordered. Monitor eschar, débridement of
wound, and range of motion. Postoperative care applies after each surgery. Maintain and assess nutritional
status.
c. Rehabilitation phase: Return to productive life and adaptation of social and physical skills may take years.
Activity 2
35. Instruct patients to conduct a complete monthly self-examination of the skin and scalp, noting moles, blemishes,
and birthmarks. Perform the examination after a bath or shower, including a head-to-toe check. Use a well-lit
room and mirrors to examine all skin surfaces. If necessary, have the patient ask a family member/significant other
to aid in the investigation. Use the ABCDE mnemonic. Contact health care provider if lesion or mole bleeds, oozes
or feels different (swollen, hard, lumpy, itchy, or tender to the touch).
36. Wear wide-brimmed hats and long sleeves. Apply sunscreens with a sun protection factor of 15 or greater
approximately 15 minutes before going into the sun and after swimming or perspiring. Reapply every 2 hours.
Avoid tanning under the direct sun at midday (10 AM to 4 PM). Do not use indoor sunlamps, tanning parlors, or
tanning pills. Some medications (e.g., oral contraceptives, antibiotics, antiinflammatories, antihypertensives,
immunosuppressives) make the skin more sensitive to the sun. Prevention methods should be used for children,
because severe sunburns in childhood greatly increase the risk for melanoma later in life.
CHAPTER 4—CARE OF THE PATIENT WITH A MUSCULOSKELETAL DISORDER
Figure Labeling
1.
See Figure 4-2 A.
Short Answer
2.
a. Support
b. Movement
c. Mineral storage
d. Hemopoiesis
e. Protection
3.
a. Motion
b. Maintenance of posture
c. Production of heat
4.
5.
Perform the 7 Ps of orthopedic assessment to establish a baseline and monitor changes in the patient’s muscular
function, bone integrity, distal circulation, and sensation:
Pain: Does it seem out of proportion to the patient’s injury? Does the pain increase on active or passive motion?
Pallor
Paresthesia or numbness
Paralysis
Polar temperature: Is the extremity cold compared with the opposite extremity?
Puffiness from edema or a hematoma
Pulselessness: A Doppler ultrasound device may be useful to determine the presence or absence of blood flow if
unable to palpate distal pulses
Treatment of sprains usually consists of rest, ice, compression, and elevation (RICE) of the affected area.
Multiple Choice
6
Answer 1, 2, 3, 5: Neurovascular assessments are made on patients with musculoskeletal trauma or damage to
nerves and blood vessels resulting from surgery, tight bandages, splints, or casts. Volkmann's contracture is a
permanent contracture (with clawhand, flexion of wrist and fingers, and atrophy of the forearm) that can occur
because of compartment syndrome. Vigilant neurovascular assessment is necessary to prevent this complication.
7. Answer 3: Alendronate (Fosamax) is prescribed for osteoporosis. Patients are encouraged to do regular weightbearing exercise to strengthen muscles, prevent bone loss and to stimulate bone formation. Calcium and vitamin D
supplements may be prescribed, and the nurse should review dietary sources for these nutrients.
8. Answer 4: Lisinopril (Zestril) is an antihypertensive medication and the combination of lisinopril and ibuprofen can
cause a hypertensive response.
9. Answer 2, 3, 4: Controlling the environment, regular exercise and a sleep diary are recommended. A hot bath and
eating just before going to bed are not recommended.
10. Answer 4: Early signs of shock are restlessness, anxiety, weakness, change of mental status, and tachycardia. The
patient may have cool, moist, pale skin. As shock progresses, hypotension, hypothermia and oliguria occur. Pain in
the muscles, bones, and joints; headaches, altered thought processes, and stiffness are symptoms of fibromyalgia.
During fat embolism, the patient may complain of chest pain, especially on inspiration and localized muscle
weakness. Spasticity, rigidity, irritability, restlessness, disorientation and stupor may occur. Deep, unrelenting,
progressive, and poorly localized pain unrelieved by analgesics is characteristic of compartment syndrome
11. Answer 1: Diarrhea, nausea, and vomiting are potential side effects of colchicine. Fluid retention and sodium
retention are side effects of adrenocorticosteroids. Seizures and dysrhythmias are side effects of meloxicam (Mobic),
which is an nonsteroidal antiinflammatory drug. Hypercalcemia and orthostatic hypotension are side effects of
teriparatide (Forteo) which is used for postmenopausal women who are at increased risk for osteoporosis
fractures or who cannot use other treatments.
12. Answer 1, 2, 4: Foods that are good sources of calcium include whole and skim milk, yogurt, turnip greens, cottage
cheese, ice cream, sardines with bones, spinach, many green vegetables, calcium-fortified orange juice, and soymilk.
13. Answer 2: The health care provider is most likely to order an x-ray examination of the ankle to rule out fracture. The
radiation exposure is minimal; however, female patients of childbearing age should always be asked about
pregnancy. Assessment of allergies and medications and past treatments are good general questions for all patients,
but in this case are less relevant to the diagnostic test that will most likely be ordered.
14. Answer 3: Loss of sensation and movement are unexpected complications that should be reported. Headache is the
most common symptom, but if correct positioning and ordered analgesics do not relieve the pain, this should also be
reported. Patients are encouraged to take fluids to flush the dye from the body. Patients are usually in a flat or semiFowler’s position for 8-12 hours; the nurse would explain the purpose of the position and initiate diversion
interventions (e.g., television, reading, listening to music).
15. Answer 2: Ankylosing spondylitis can affect the cardiovascular and respiratory systems. Inflammatory bowel
disease occurs in about 3-10% of patients. Back pain and stiffness, weight loss, vision change, and fatigue are
common. The 7 Ps could be used but apply more to assessment of extremities. Mental status and urination should
not be directly affected.
16. Answer 3: The patient is describing the symptoms of gout; thus, the nurse would do a dietary history to include
specific questions about alcohol, organ meats, anchovies, yeast, herring, mackerel, or scallops, because foods high in
17.
18.
19.
20.
21.
22.
23.
24.
25.
26.
27.
28.
29.
30.
purines worsen gout. Patients with ankylosing spondylitis should be asked about bowel changes. All patients
should be asked about exercise routines. Jaw tension, excessive fatigue, or anxiety would be more typical for
patients with fibromyalgia.
Answer 1: Osteomyelitis is an infection of the bone. Drainage precautions are initiated, because the wounds
frequently require débridement, irrigation, and sterile dressing changes. Ambulating may be restricted because the
affected part is usually rested. Patients with arthritis or fibromyalgia are more likely to have trouble moving in the
early morning. Ice packs are more appropriate for patients with sprains or strains; sometimes for patients with
arthritis.
Answer 1, 2, 4, 5: Coughing and deep-breathing, clear liquids with transition to a regular diet, assessing ability to
use assistive devices, and monitoring IV fluids and antibiotics would be included in the care of the patient who had
unicompartmental knee surgery. The patient would not have a cast and intraarticular injections of corticosteroids
would be given by the health care provider for rheumatoid arthritis.
Answer 2: The nurse’s first action would be to assess for signs/symptoms of hypovolemic shock. An increase in
pulse is an early sign. A decrease in blood pressure comes later. The nurse could also look at the urinary output, but
the most useful piece of data is to know output per hour. Reassurance and visitors are appropriate if the patient is
physically stable and needs additional emotional support.
Answer 2: Pain is a primary symptom of compartment syndrome or infection. In addition, pain is a subjective
symptom that the child will have to report to parents. Capillary refill, cast care and maintenance are important, but
the parents can be given written information about these topics. Fiberglass casts do not degrade if they get wet, but
drying them out can be time-consuming.
Answer 4: The head of the bed should not be elevated past 45 degrees to a avoid acute flexion on the device. The
other actions are part of the postoperative care.
Answer 1: Bedrest is typically for the first 24 hours. The other comments are correct.
Answer 2: When a person falls, the natural instinct is to extend the arms out to break the fall. This results in a Colles’
fracture, which is a fracture of the distal portion of the radius within 1 inch of the wrist joint. A head-to-toe
assessment always gives good information, but the obvious injuries should be addressed first in this “field”
situation. Mental status examination would be the priority if the patient could not relate details of the fall (e.g., loss
of consciousness because of a cardiac, neurologic, or metabolic event). Based on the patient’s current status, the
environmental assessment should be performed after other potential injuries are assessed.
Answer 1: The patient has signs and symptoms of a pelvic fracture and hemorrhage is the most life-threatening
complication. Hemoglobin and hematocrit are laboratory indicators of blood loss. Blood type and Rh are important
if the patient needs emergency surgery. Urinalysis and stool for occult blood are performed because of the position
of the bladder and the colon in the pelvic area.
Answer 4: The nurse performs the assessment first. Based on the assessment findings, the nurse may decide to use
the other options.
Answer 2: Volkmann’s contracture is a permanent contracture that can result from undetected and untreated
compartment syndrome. The result is clawhand with flexion of the wrist and hand and atrophy of the forearm. The
nurse would assess the patient’s abilities to perform activities of daily living. The other options are actions that
should have been performed during the patient’s initial injury and treatment.
Answer 2: The arterial blood gases are within normal limits. The patient with a long bone fracture is at risk for fat
embolism, but the occurrence is relatively rare. However, respiratory failure is the most common cause of death
associated with fat embolism, so the nurse would continue to monitor the patient.
Answer 2: Frequent position changes and stretching hands are preventive measures for carpal tunnel syndrome.
Warm packs will worsen the inflammation and edema. Suggesting use of medication, even over-the-counter
medications, is not advised, especially because the health care provider has not evaluated the medical condition.
Wrapping the wrist may help a bit, but the health care provider is likely to recommend the use of a commercial
splint.
Answer 3: Patients who have had a laminectomy are at risk for a paralytic ileus; therefore, the nurse would first
assess for possible bowel obstruction.
Answer 4: An elevated serum alkaline phosphatase signals osteogenic sarcoma or other bone disorders (liver disease
is also associated with elevated alkaline phosphatase). Phantom limb pain occurs after amputation for some
individuals. Fibromyalgia has a variety of symptoms, but the pain tends to be in the muscles and in the low back.
Compartment syndrome is the result of excessive pressure within the fascial compartments, usually caused by a cast
or dressing, but can also be caused by a crushing injury. (Note to student: Compartment syndrome is frequently
associated with the extremities, but can also occur in the abdomen.)
Critical Thinking Activities
Activity 1
31. Genetic and environmental factors, such as small bone structure and lack of exercise, can contribute to the rate of
bone loss. Individuals most at risk for developing osteoporosis are small-framed, white (European descent) or Asian
race. Smoking and alcoholism also increase the risk. Medical conditions associated with an increased development
of the disease include hyperthyroidism, chronic lung disease, cancer, inflammatory bowel disease, alcoholism, and
vitamin D deficiency. Medications that are linked to the development of osteoporosis include steroids,
anticonvulsants, immunosuppressant therapies, and heparin. Diets low in calcium or high in caffeine and protein
are also implicated.
Nursing interventions are aimed at preventing further bone loss and fractures. Teach the patient to include milk
and dairy products in the diet. Use vitamin D supplements as prescribed. Food and beverages that contain caffeine
also contain phosphorus, which contributes to bone loss. Encourage smoking cessation. Safety measures, such as
side rails, handrails, bedside commodes with seat elevators, and rubber mats in showers can help prevent falls in
older adults. Encourage ambulation to prevent further loss of bone substance. Encourage weight-bearing exercise to
increase bone density.
Activity 2
32. Fibromyalgia syndrome is not life-threatening, but 50% of patients report that they have trouble completing
activities of daily living. There is a wide range of symptoms, such as aches, fatigue, cognitive difficulties, problems
sleeping, anxiety, depression, and tingling sensations. Symptoms can overlap with chronic fatigue syndrome. There
are no specific diagnostic tests; thus, an exclusion approach is used, and the diagnosis could take years. FMS is hard
to treat, and many will have trouble achieving remission.
Activity 3
33. Women are at greater risk for hip fracture due to their increased occurrence of osteoporosis and longer life
expectancy compared with men. This woman is thin and therefore has inadequate local tissue to absorb shock.
Climbing stairs to the second floor requires coordination and balance that change with age. The loose rugs and
clutter are hazardous and low light impairs vision. Bending to pet the dog or having him jump on her can put her
off balance; the physiologic changes of aging result in decreased joint flexibility and muscular strength. The cane,
walker, and eyeglasses could help prevent falls; the nurse could help the woman to devise methods to keep track of
these items.
CHAPTER 5—CARE OF THE PATIENT WITH A GASTROINTESTINAL DISORDER
Figure Labeling
1.
See Figure 5-1.
Short Answer
2.
a. Salivary amylase (ptyalin), initiates carbohydrate metabolism.
b. Lysozyme protects the mucous membrane from infections and the teeth from decay by destroying bacteria.
c. After food has been ingested, continued secretion of saliva, cleanses the mouth.
3.
a. Completion of absorption of water
b. Manufacture of certain vitamins (such as vitamins K and B7)
c. Formation of feces
d. Expulsion of feces.
4. Liver functions: produces bile, manages blood coagulation; metabolizes proteins, fats, and carbohydrates;
manufactures cholesterol; manufactures albumin (for normal blood volume); filters out old red blood cells and bacteria;
detoxifies poisons (alcohol, nicotine, drugs); converts ammonia to urea; provides the main source of body heat at rest;
stores glycogen for later use; and activates vitamin D.
Multiple Choice
5. Answer 1 : Research has revealed that nonpathologic bacteria live safely in the appendix until they are needed for
digestion. In addition, the appendix houses immune system cells and tissue.
6. Answer 4: Bacteria in the large intestine are responsible for the synthesis of vitamin K, which is needed for normal
blood clotting. Vitamin A is needed to synthesize a photosensitive pigment, which is necessary for color vision.
Vitamin D is necessary for bone formation. One of the functions of the B vitamins is to facilitate intestinal
absorption.
7. Answer 3: Fluids are encouraged to help flush barium from the gastrointestinal tract. Hardened barium can cause an
impaction.
8. Answer 2: For esophagogastroduodenoscopy, the patient is given an IV sedative and the pharynx is anesthetized by
spraying it with lidocaine hydrochloride (Xylocaine). Following the procedure, the gag reflex is assessed by placing a
tongue blade to the back of the pharynx. The nurse must monitor vital signs, oxygen saturation, oral bleeding, blood
in the stool, and signs and symptoms of perforation and hypovolemic shock. For capsule endoscopy, the patient
needs reassurance that the capsule will pass with feces. For a barium swallow, the patient needs to be advised about
the pale color of stools and to increase fluid intake to prevent impaction. The three tests for H. pylori are noninvasive
and the patient would be advised about the approximate time to expect the results.
9. Answer 4: Subjective symptoms of gastroesophageal reflux disease can mimic cardiac problems. Even if the patient
suspects heartburn, the nurse would maintain a high index of suspicion until cardiac disorders are ruled out by the
health care provider. The other rationale are also appropriate. The nurse would assess before giving medication or
calling the health care provider. If there is significant hemorrhage from the upper gastrointestinal tract, it is likely
that the patient would vomit blood.
10. Answer 2: Hemorrhagic colitis can develop after eating meat (especially ground meat) that has been contaminated by
E. coli—serotype O157:H7. This condition can be life threatening, particularly for the very young and very old.
Acute gastritis related to ingestion of alcohol is not contagious. Patient may need treatment if gastrointestinal
bleeding occurs. Burning sensation accompanied by regurgitation is characteristic of gastroesophageal reflux
disease, which is treatable with medicine and lifestyle modifications. Patients who have celiac disease experience
abdominal bloating and muscle aches after eating foods that contain gluten.
11. Answer 2, 3, 4, 5: Injury, trauma, or disruption of the anal sphincter can result in fecal incontinence. Spinal cord
lesions can result in loss of conscious control of defecation. Normal changes that occur with aging are usually not
significant enough to cause incontinence. Voluntary inhibition of defecation is learned in childhood as a means to
control emptying of the rectum.
12. Answer 2: Musculature of the bowel contains its own nerve centers that respond to distention through peristalsis.
Therefore, even when the patient has motor paralysis, reflex defecation often persists or can be stimulated. Bowel
training is a better long-term option; the other options could be considered as interim measures until bowel control
is achieved.
13. Answer 1: Biofeedback training has been proven effective with alert, motivated patients who have motility disorders
or sphincter damage that causes fecal incontinence. The patient learns to tighten the external sphincter in response to
rectal distention. Patients with structural damage may be candidates for surgical repair. For patients with
dementia, timed toileting and habit training is useful. Incontinence should resolve with removal or dislodgement of
the impaction.
14. Answer 3: High-fiber foods facilitate defecation. Fluids should also be encouraged.
15. Answer 2: Sucralfate (Carafate) acts by coating the gastric mucosa. This decreases the absorption of other drugs;
therefore other medications are separated by 2 hours. Misoprostol (Cytotec) is contraindicated during pregnancy.
Cimetidine (Tagamet) increases the serum levels of oral anticoagulants, theophylline, phenytoin, some
benzodiazepines, and propranolol. Diphenoxylate with atropine (Lomotil), dimenhydrinate (Dramamine), atropine,
scopolamine, hyoscyamine, dicyclomine, and clidinium (Donnatal, Bentyl) are just a few of the drugs that can cause
sedation.
16. Answer 1: Intrinsic factor (a substance secreted by the gastric mucosa) is produced to allow absorption of vitamin
17.
18.
19.
20.
21.
22.
23.
24.
25.
26.
27.
28.
29.
30.
B12. Pernicious anemia can develop because of vitamin B12 deficiency. Patients with a partial gastrectomy should
have a blood serum vitamin B12 level measured every 1 to 2 years so that replacement therapy of vitamin B 12 via a
monthly injection or via nasal route weekly can be instituted before anemia appears. Hemoglobin and hematocrit
would be monitored when blood loss is suspected. Iron dextran can be given for anemia associated with blood loss,
for example in Crohn’s disease. Increasing fruits and vegetables and decreasing red meat and fat is good general
advice but is inadequate to address the patient’s risk for pernicious anemia.
Answer 2, 3, 4, 5, 6: Stomach carcinogenesis probably begins with a nonspecific mucosal injury as a result of aging;
autoimmune disease; or repeated exposure to irritants such as bile, antiinflammatory agents, or smoking. Other
factors include history of polyps, pernicious anemia, hypochlorhydria (deficiency of hydrochloride in the stomach’s
gastric juice), chronic atrophic gastritis, and gastric ulcer. Because the stomach has prolonged contact with food,
cancer in this part of the body is associated with diets that are high in salt, smoked and preserved foods (which
contain nitrites and nitrates), and low in fresh fruits and vegetables.
Answer 2, 4, 5: The onset of Crohn’s disease is usually insidious, with nonspecific complaints such as diarrhea,
fatigue, abdominal pain, and fever. As the disease progresses, the patient experiences weight loss, malnutrition,
dehydration, electrolyte imbalance, anemia, and increased peristalsis.
Answer 2, 4, 5: The patient should be kept on bedrest and receive no foods or fluids by mouth. Vital signs should be
monitored because there is a risk for peritonitis. Antibiotics can be given if perforation is suspected or may be given
as a preoperative medication. Enemas and heating pads should not be used because of increased risk for peritonitis.
Antacids are unlikely to offer relief to this patient.
Answer 3: The nurse would assess the abdominal pain, check the vital signs, and assess for other symptoms of
hypovolemic shock. Other symptoms of perforation would include melena, oral bleeding, and guarding.
Answer 1: The pill camera passes through the gastrointestinal system in 2-3 days. There is no need to retrieve the
camera and problems with passing the device or change in stool are not expected.
Answer 4: During the procedure, mild hydrochloric acid is administered through the nasogastric tube. If pain
increases, then the test is considered positive. Relief of pain by nitrates is more associated with anginal pain.
Antacids could provide some relief and are used in the treatment of reflux and gastritis. Decompression of the
stomach can provide relief; for example, in the case of obstruction or pancreatitis.
Answer 1: Barium is a contrast medium that can interfere with visualization during a colonoscopy or in the
interpretation of the flat plate and ova and parasite examinations.
Answer 2: Removing the plaques can cause pain and bleeding. The other actions are correct in the care of oral
candidiasis.
Answer 3: For lesions that do not heal within 2-3 weeks, the neighbor should seek medical attention. Diluted
hydrogen peroxide can be used for candidiasis or halitosis. Lipstick or lip balm that includes sunscreen and
consuming fruit and vegetables are good preventive measures, but are inadequate to address the existing lip lesion.
Answer 2: The conservative approach focuses on modification of lifestyle, which includes avoiding foods and
beverages that contribute to discomfort, smoking and alcohol cessation, losing weight, sleeping with head elevated,
and not lying down immediately after eating. Medications are also used in a step-up fashion. Nissen fundoplication
is a surgical procedure that would be used if medical therapies are not successful. Barrett’s esophagus is associated
with long-term untreated gastroesophageal reflux disease; it is considered precancerous and requires endoscopy
and biopsy every 1-3 years.
Answer 3: Perforation is the most lethal complication of peptic ulcer disease (PUD) because of peritonitis. An
elevated white blood cell count will accompany this potentially lethal infection. Fecal assay antigen and occult blood
are used to diagnosis PUD. Pain during the hydrochloric test is used to diagnose gastroesophageal reflux disease.
Answer 2: The patient is describing symptoms of dumping syndrome which occurs in approximately one-third to
one-half of patients who have surgery for peptic ulcer disease. Symptoms are usually triggered by a bolus of
hypertonic food. The other questions could be used to gather additional information.
Answer 3: The use of antidiarrheals is not recommended because the body is trying to rid itself of the E. coli
pathogen. The health care provider could order antidiarrheals if the fluid loss is relentless. Oral fluids are the first
choice, but IV fluids can be ordered if the patient is having trouble with oral fluids or to replace initial fluid loss.
Contact isolation would be appropriate to prevent the spread to others.
Answer 4: C. difficile is not destroyed by antiseptic hand rub, so soap and water are required for adequate hand
hygiene. The other options are not part of contact isolation or needed for the care of this patient.
31.
32.
33.
34.
35.
36.
37.
38.
39.
40.
41.
42.
Answer 4: Foods containing wheat, rye, and barley should be avoided. Foods that commonly contain these
ingredients include: cereal, pasta, and many processed foods. Plain meat and fish, fruits, vegetables, and rice can be
included into the diet since they do not contain gluten.
Answer 3: First the nurse acknowledges feelings and then assesses what the patient understands about the disease
and the diagnostic process. Based on the assessment, the nurse may decide to use the other options. (Note to student:
Recall principles of therapeutic communication by starting where the patient is emotionally; acknowledge and
encourage expression of feelings.)
Answer 1: With severe diarrhea, the body loses sodium, potassium, calcium, and bicarbonate. Hematocrit levels are
likely to be elevated because of fluid loss. A fecal sample is likely to show blood because of irritation to the mucosa.
Liver function tests should not be relevant to this condition.
Answer 2: First the nurse uses therapeutic communication to help the patient control anxiety by encouraging the
expression of feelings about the procedure and other concerns. If the patient leaves, there is no opportunity for
other interventions. Based on the assessment of concerns, the nurse may decide to use the other options.
Answer 1: Crohn’s disease causes ulceration with fistula formation that can connect the colon with the urinary tract.
The urine of patients with suspected appendicitis will be tested to rule out urinary infection as a source of the pain.
Patients with ulcerative colitis could develop urinary tract infections related to improper hygiene of the perineal
area; thus staff and patients should be aware to clean and wipe from front to back. Peptic ulcer disease should not
contribute directly to urinary tract infections.
Answer 4: Side-lying with knees flexed (fetal position) is preferred because this decreases the strain on the
abdominal wall.
Answer 2: For acute diverticulitis, the patient is likely to receive nothing by mouth. The other actions are correct.
Answer 1: The nurse recognizes the potential for peritonitis; however, additional assessment with vital signs should
be performed before sitting the patient in semi-Fowler’s position (blood pressure could be low, and pulse elevated
because of shock) or notifying the health care provider who will ask about the last set of vital signs. An as needed
pain medication is not appropriate if peritonitis is suspected. (Remember to apply the nursing process; the first step
is assessment.)
Answer 1, 2, 4, 6: Monitoring vital signs, pain, bowel sounds, fluid balance, and drainage and bleeding are
appropriate care. The patient should turn, cough, deep-breathe, and be encouraged to ambulate. The indwelling
catheter should be removed as soon as possible to prevent infection and to allow adequate time to assess the
patient’s ability to void. Suction should be temporarily discontinued during ambulation.
Answer 3: Increasing fluid intake and a high-fiber diet decrease the likelihood of constipation; straining at stool can
cause hemorrhoids. Suggesting use of hydrocortisone creams or rubber-band ligation is the responsibility of the
health care provider.
Answer 1: Loud frequent high-pitched bowel sounds are heard early. The other signs/symptoms occur later.
Answer 2: Rectal bleeding is the most common sign of colorectal cancer. The other signs/symptoms will occur later.
Critical Thinking Activities
Activity 1
43. Dietary instructions include: (1) eat four to six small meals daily; (2) follow a low-fat, adequate-protein diet; (3) reduce
intake of chocolate, tea, and other foods and beverages that contain caffeine; (4) limit or eliminate alcohol intake; (5)
eat slowly, and chew food thoroughly; (6) avoid evening snacking, and do not eat for 2 to 3 hours before bedtime; (7)
remain upright for 1 to 2 hours after meals when possible, and never eat in bed; (8) avoid any food that directly
produces heartburn; and (9) reduce overall body weight if needed.
Lifestyle modifications include: (1) smoking cessation, (2) avoid constrictive clothing over the abdomen, (3) avoid
activities that involve straining, heavy lifting, or working in a bent-over position. (4) elevate head of the bed at least 6
to 8 inches for sleep, using wooden blocks or a thick foam wedge.
Activity 2
44. This patient is young, and though the symptoms are early and relatively mild they are interfering with his lifestyle.
He has already undergone a colonoscopy, which is atypical for a person his age and he may have difficulty
complying with the dietary recommendations. The disease is chronic, and he will face many exacerbations and
remissions. It is likely that he will eventually need surgery (75% chance) with an ostomy. There is no known
treatment to keep him in remission and recurrence rate is high for patients under age 25.
Activity 3
45. Older adults experience loss of teeth and resultant use of dentures can interfere with chewing. Dysphagia is
common and may be caused by changes in the esophageal musculature or by neurologic conditions. Hiatal hernias
and esophageal diverticula are caused by changes in musculature of the diaphragm and esophagus. Decreased
secretion of hydrochloric acid results in an increased incidence of pernicious anemia and gastritis. Peptic ulcers are
common, but often the symptoms are vague and go unrecognized until there is a bleeding episode. Medications
such as aspirin, nonsteroidal antiinflammatory drugs, and steroids that are taken for the chronic degenerative joint
conditions can contribute to ulcers. Diverticulosis and diverticulitis contribute to malabsorption of nutrients.
Inactivity, changes in diet and fluid intake, and medications can contribute to constipation.
Activity 4
46. a. Presence of distention, visibility of peristaltic waves, vomiting, tenderness, guarding behaviors, presence and
characteristics of bowel sounds
b. Abdominal x-rays, CT scans, sigmoidoscopy or colonoscopy may be used to confirm the presence of an
intestinal obstruction. Hematologic studies may be used to assess the degree of impact of the obstruction. These
blood studies include electrolyte levels and hemoglobin and hematocrit readings.
c. Removal of gas and fluid, correction of electrolyte imbalances, relief or removal of the obstruction
d. The manifestations of mechanical and nonmechanical intestinal obstructions are similar. The primary difference
between the types is the underlying cause. Nonmechanical intestinal obstructions result from a neuromuscular
or vascular disorder. Mechanical obstructions are caused by a physical occlusion in the intestinal tract.
Regardless of the cause of the obstruction, gastric contents cannot pass through the gastrointestinal tract.
CHAPTER 6—CARE OF THE PATIENT WITH A GALLBLADDER, LIVER, BILIARY TRACT, OR EXOCRINE
PANCREATIC DISORDER
Short Answer
1.
Modifiable risk factors include: obesity, cigarette smoking, exposure to chemical carcinogens, diets high in red
meat and pork (especially processed meat such as bacon), fat, and coffee. Nonmodifiable factors include:
genetics, and African-American male. Disease processes associated with increased risk include: diabetes mellitus,
cirrhosis, and chronic pancreatitis.
Multiple Choice
2.
Answer 1: Jaundice is a yellow discoloration of body tissues that is caused by high blood levels of bilirubin. It is
visible when the total serum bilirubin exceeds 2.5 mg/dL. Pallor is seen in patients with anemia. Cyanosis is
associated with decreased oxygenation. Cherry red color is seen with carbon monoxide poisoning.
3.
Answer 2: A low serum albumin can be caused by protein-calorie malnutrition. Normal range for albumin is 3.5 to 5
g/dL. This patient's albumin level is only slightly below normal. Other causes include nephrotic syndrome, ascites,
liver disease and increased capillary permeability (e.g., severe burns).
4.
Answer 4: During the needle liver biopsy there is a chance that blood or bile can leak into the peritoneal cavity or
the needle can enter the chest cavity, so pneumothorax is possibility. The signs and symptoms of pneumothorax
are shortness of breath, decreased oxygen saturation and chest pain. Ill effects are not anticipated for the other
diagnostic tests, although, patients can have delayed allergic reactions to contrast media if it is used during
computed tomography.
5.
Answer 3: During endoscopic retrograde cholangiopancreatography (ERCP), the patient must lie motionless on the
table for 1-2 hours and a fiberoptic scope is orally inserted. The health care provider may not realize that the patient
will not be able to cooperate with instructions or tolerate lying motionless for 1-2 hours. The other tests require
specimen collection, which may also be traumatic for the patient, but the surroundings will be familiar, and the
collection process is relatively faster.
6.
Answer 2: Ascites is a result of portal hypertension, hypoalbuminemia, and hyperaldosteronism. Asterixis is a
hand-flapping tremor seen in patients who are acutely ill with hepatic encephalopathy. Flatulence is excess
formation of gases in the stomach or intestine; it can be caused by eating certain foods and it occurs in many
gastrointestinal disorders. Cachexia is a physical wasting that can be concurrent with or preceded by anorexia (loss
of appetite).
7.
Answer 4: Early subject symptoms include the patient's description of flulike symptoms (loss of appetite, nausea
and vomiting, general weakness, and fatigue). The other signs/symptoms occur later.
8.
Answer 2: The cirrhotic liver cannot absorb vitamin K or produce the clotting factors VII, IX, and X. This results in
bleeding tendencies. Bleeding from any source should be investigated but change of mental status after head trauma
warrants investigation for possible intracranial bleeding (alcoholics are especially prone to intracranial bleeding
even with minor trauma). For the other problems with bleeding, the nurse may opt to call the health care provider
to obtain an order for coagulation studies, or the patient may need additional teaching about self-care measures for
bleeding tendencies.
9.
Answer 1: The damaged liver cannot metabolize protein; therefore, protein intake may result in an elevation of
blood ammonia levels. Disorientation and abnormal behaviors and speech patterns can occur with increased
ammonia levels.
10. Answer 1, 2, 3, 4, 6: People who have high risk for exposure (needlesticks, body fluid exchange, geographic areas
where the incidence is higher) to the virus should receive the hepatitis B vaccine. Children under the age of 18 are
routinely vaccinated. Elderly residents in long-term care facilities should assessed for risk and then the decision is
made on a case-by-case basis.
11. Answer 1: Lactulose has a cathartic action and is used to prevent ammonia build up. Ammonia is toxic to the brain
and high levels will manifest as neurologic signs and symptoms that progress from inappropriate behavior,
disorientation, asterixis, and twitching of the extremities to stupor and coma.
12. Answer: 1,2,3: Health care personnel should wear gown and gloves when care requires direct contact with the
patient's body fluids. Standard precautions, such as hand hygiene and sharps safety are always used. Additional
13.
14.
15.
16.
17.
18.
19.
20.
personal protective equipment can be used, as needed, for increased exposure to body fluids. For example, masks or
eye shields can be used during nasogastric tube insertion or endotracheal intubation because of possible vomiting,
coughing or sneezing.
Answer 3: Before discharge, patients should be able to eat without difficulty and walk, and should have no
abdominal distention, evidence of bleeding, or bile leakage. Mild shoulder pain is caused by diaphragmatic
irritation secondary to abdominal stretching or residual carbon dioxide. T-tubes are inserted during open
cholecystectomy; in laparoscopic cholecystectomy, the gallbladder is removed through one of four small punctures
and the punctures are covered with band aids. As long as the patient is able to eat, temporary loss of appetite is not
a concern.
Answer 4: Patient should exhale and hold breath while the needle is being inserted. This allows the health care
provider to insert the needle between the sixth and seventh or eighth and ninth intercostal spaces and into the liver.
Answer 1: The purpose of the T-tube is to allow the bile to drain out. Initially, up to 500 mL of drainage would be
considered an expected outcome. The flow should decrease over time. Inflammation, pain, and bleeding are not
expected findings.
Answer 4: The pain is expected because of diaphragmatic irritation and residual carbon dioxide. The appropriate
intervention is to give an analgesic.
Answer 2: The level of lipase is more specific for diagnosing acute pancreatitis. Low albumin, increased glucose, and
elevated amylase are likely to accompany the diagnosis of pancreatitis; however, changes in albumin, glucose, and
amylase can be associated with many other disorders.
Answer 1: Hepatitis E is most often seen in southeastern and central Asia, the Middle East, Africa, and Mexico.
Drinking water from questionable sources and eating raw shellfish increases the risk for hepatitis E. High-risk
sexual behaviors and sharing needles are sources of hepatitis B and C. Unsafe tattoo methods are associated with
hepatitis G
Answer 2: The number of tablets or ingestion of fatty food just before the test could alter the outcome. Also,
vomiting and diarrhea can alter the absorption of the dye. Laxatives and enemas are usually not required. Amount
of fluid should not affect examination; however, fat in the fluids (i.e., whole milk) could be a factor.
Answer 1: For a pregnant woman, ultrasound offers a safe option. Oral cholecystography and intravenous
21.
22.
23.
24.
25.
26.
27.
cholangiography and computed tomography require exposure to x-rays.
Answer 3: There are no special instructions for care after a hepatobiliary iminodiacetic acid scan; verbally reassuring
the unlicensed assistive personnel is appropriate, because he/she may not be familiar with what happens during the
diagnostic procedure. The amount of radioisotope is very minimal, so the dosimeter is not required. (Note to student:
Certain units or jobs may require that all personnel wear dosimeters all the time.) The isotope is given
intravenously, but bleeding is not an expected side effect of the procedure.
Answer 2: The needle liver biopsy is an invasive test that creates a potential for hemorrhage, shock, peritonitis, and
pneumothorax; thus, frequent assessment of vital signs is required. The serum ammonia test is accomplished by
drawing a blood sample. Oral cholecystography and radioisotope liver scan do not require any care beyond routine
assessment after returning from the procedure.
Answer 3: The purpose of a soft toothbrush with gentle brushing action is a precaution initiated when patients are at
risk for bleeding. In this case, the cirrhotic liver cannot absorb vitamin K or produce the clotting factors VII, IX, and
X.; this contributes to bleeding.
Answer 1, 2, 4: Preoperative patients need to learn about coughing and deep-breathing and would be ideal
candidates for the student. The patient with chronic hepatitis would benefit because of prolonged bedrest. The
patient with esophageal varices should not be encouraged to cough because of the potential for rupture. The patient
with acute pancreatitis needs to cough and deep-breathe, but this patient is less than ideal for a first-semester
student, because acute pancreatitis causes severe pain and the patient may have less tolerance for the novice.
Answer 3: Hepatic encephalopathy is a type of brain damage caused by liver disease and consequent ammonia
intoxication. The other tests are also included in the general diagnosis of liver disease.
Answer: 1.4 mL
155 lbs ÷ 2.2 = 70.45, rounded to 70 kg
70 kg × 0.02 mL/kg = 1.4 mL
Answer 4: If the patient knows that the procedure will provide relief for noxious symptoms, he/she is more likely to
cooperate. Nasogastric tube insertion is extremely uncomfortable but giving pain medication does not alleviate the
sensations of tearing or gagging. An antianxiety medication may be more effective in this case. Having the most
experienced nurse insert the tube is a good strategy for an anxious patient, but he/she must still agree to cooperate.
Calling the health care provider is also appropriate if the patient is determined to leave the hospital.
Critical Thinking Activities
Activity 1
28. a. Infection and rejection of the organ
b. Respiratory complications (pneumonia, atelectasis, pleural effusions), hemorrhage, infection, and electrolyte
imbalances
c. The patient will be closely observed for signs of rejection (e.g. pain at site, fever, flu-like symptoms). There will
be medications to reduce the likelihood of rejection. Cyclosporine is an effective immunosuppressant drug.
Other immunosuppressants used include azathioprine (Imuran), corticosteroids, tacrolimus (Prograf),
mycophenolate mofetil (Cellcept), and new agents including the interleukin-2 receptor antagonists basiliximab
(Simulect) and daclizumab (Zenapax).
d. Coughing and deep-breathing exercises, monitoring neurologic status, signs of hemorrhage, input and output,
assessment of drainage from Jackson-Pratt drain, nasogastric tube, and T-tube. The nurse should monitor for
signs and symptoms of infection and rejection (e.g. pain at site, fever, flu-like symptoms [loss of appetite, nausea
and vomiting, general weakness, and fatigue]).
Activity 2
29. a. Cholelithiasis
b. Increased heart and respiratory rates, diaphoresis, elevated temperature, elevated leukocyte count, mild
jaundice, and steatorrhea
c. Fecal studies, serum bilirubin tests, ultrasound of the gallbladder and biliary system, hepatobiliary
iminodiacetic acid scan, or operative cholangiography may be done. Ultrasound of the gallbladder is highly
accurate in diagnosing cholelithiasis.
Activity 3
30. a. Symptoms can be vague and insidious; therefore, cancer is usually well-established before it is diagnosed and
life expectancy can be 4 to 6 months after diagnosis. The patient may have to undergo many diagnostic tests and
will then be told that tumors are inoperable. The pain is likely to be significant.
The patient may express regret because of failure to modify lifestyle (smoking; obesity; diet that includes red
meat, fat, and coffee contribute to risk for pancreatic cancer), fear related to death, frustration related to
intensive diagnostic testing and treatments that provide little hope for cure. The patient will be dealing with
severe pain while having to face loss of social, work, family, and community roles.
b. The nurse is aware that the patient faces many challenges. Active listening encourages expression of fears and
concerns. Give information as needed to decrease anxiety. Expert care and anticipating needs also helps to
decrease the patient’s anxiety. Refer to social services and support groups as appropriate.
CHAPTER 7—CARE OF THE PATIENT WITH A BLOOD OR LYMPHATIC DISORDER
Short Answer
1.
The blood performs three critical functions. First, it transports oxygen and nutrition to the cells and waste products
away from the cells, and it transports hormones from endocrine glands to tissues and cells. Second, it regulates the
acid-base balance (pH) with buffers, helps regulate body temperature because of its water content, and controls the
water content of its cells as a result of dissolved sodium ions. Third, it protects the body against infection by
transporting leukocytes and antibodies to the site of infection and prevents blood loss with special clotting
mechanisms.
2.
The lymphatic system has three basic functions: (1) maintenance of fluid balance, (2) production of lymphocytes,
and (3) absorption and transportation of lipids from the intestine to the bloodstream.
3.
Lymph nodes (glands) have two functions: (1) to filter impurities from the lymph and (2) to produce lymphocytes
(white blood cells).
4.
The spleen: (1) has a major role in homeostasis by destroying worn-out or defective red blood cells (RBCs); (2) is a
reservoir for blood; (3) forms lymphocytes, monocytes, and plasma cells; (4) houses white blood cells in the lining of
the hollow cavities within the spleen; (5) produces RBCs before birth (the spleen is believed to produce RBCs after
birth only in cases of extreme hemolytic anemia).
Table Activity
5.
See Table 7-1 for additional information.
Blood Test
Normal Values
Red blood cells (RBCs)
Males: 4.7-6.1 million/mm3
Females: 4.2-5.4 million/mm3
Hemoglobin
Males: 14-18 g/dL
Females: 12-16 g/dL
Hematocrit
Males: 42%-52%
Females: 37%-47%
Platelet count
150,000-400,000/mm3
White blood cells (WBC) actual cell count
5000-10,000/mm3
Prothrombin time (PT)
11-12.5 seconds
International Normalized Ratio (INR)
0.7-1.8
Partial thromboplastin time (PTT)
60-70 seconds
Multiple Choice
6. Answer 4: Blood type AB is the universal recipient blood because it contains neither anti-A nor anti-B antibodies;
therefore, donor red cells containing A or B antigens do not cause clumping. If Rh-negative recipient gets Rhpositive blood, anti-Rh antibodies develop. Any subsequent transfusions with Rh-positive blood will cause a severe
reaction. If Rh-positive recipient gets Rh-negative blood, no antibodies are produced. Ordinarily all blood is typed
and cross-matched for each recipient.
7.
Answer 3: During an infection, the lymph nodes collect foreign matter, such as bacteria and become swollen. The
nurse also detects a possible fever, because the patient's skin is hot. Swelling of the lymph nodes and fever can also
occur in other disorders, such as cancer. If infection is ruled out, the health care provider will order diagnostic tests
to determine etiology of symptoms.
8.
Answer 2: The sternum is the site that has the greatest risk for penetration of underlying organs during a bone
marrow aspiration or biopsy. The posterior iliac crest is the preferred site.
9.
Answer 3: An increase in the pulse is an early sign and is a compensatory mechanism for blood loss or low volume
states, such as dehydration. Scant urine output is a compensation, but the body is trying to preserve the heart and
brain at the expensive of the kidneys. Hypotension and change of mental status will occur if the blood loss
continues.
10. Answer 1: Pain, redness and swelling suggest venous thrombosis of the lower leg. The major cause of morbidity
and mortality from polycythemia vera is thrombosis. The other signs/symptoms may also manifest in polycythemia
vera.
11. Answer 4: All of these lunch trays contain some of the foods that supply nutrients for erythropoiesis. But option 4
contains the biggest variety of foods that offer the nutrients. See Box 7-3 for additional information
12. Answer 1: When patients are dehydrated, the hemoglobin and hematocrit appear higher than normal. Restoring
fluid balance will yield normal results for hemoglobin and hematocrit. Platelet counts and prothrombin time should
not be affected.
13. Answer 3: Bandemia is seen in patients who have serious bacterial infections, so the nurse is aware of the need to
monitor for development of sepsis, which could lead to septic shock. Conditions such as dehydration or
polycythemia vera increase the risk for venous thrombosis. Thrombocytopenia is a reduction of platelets. The
basophils are involved in allergic response.
14. Answer 3: If the mother is Rh-negative the nurse would conduct further assessment (i.e., pregnancy history, father's
blood type, or previous blood transfusions.) If the mother is Rh-negative and the father is Rh-positive, anti-D
antibodies may exist from a previous pregnancy, miscarriage, ectopic pregnancy, or transfusion. In subsequent
pregnancies, if the baby is Rh-positive, hemolytic disease (in the newborn) could be triggered by the presence of the
mother’s anti-D antibodies.
15. Answer 1: Some Jehovah’s Witnesses will accept volume expanders (colloids) and autologous blood. The health care
team can administer blood to children without the consent of parents according to the US Supreme Court. It is
however within the rights of a responsible and coherent adult to refuse treatment.
16. Answer 4: The unlicensed assistive personnel (UAP) can assist the patient with self-care activities and toileting, but
the nurse must assess the patient’s limitations and give the UAP specific instructions. The UAP might apply oxygen
if there was a true emergency, but generally the patient’s shortness of breath should be reported to and assessed by
the nurse. Teaching the visitors and patients about limitations and designing an appropriate visit schedule should be
done by the nurse with consideration of the patient’s wishes and his/her limitations.
17. Answer 3: Subtle changes in behavior such as restlessness or anxiety are considered early signs. Orthostatic blood
pressure is manifest after patient loses 1000-1500 mL of blood. Decreased red cell count may not be evident in the
early stages. Decreased urine output is a compensatory mechanism that indicates that blood is being shunted away
from the kidneys in order to preserve the brain and heart.
18. Answer 3: The patient has a risk for internal bleeding (risk for hypovolemic shock) and peritonitis (risk for septic
shock). If the pain is worse, the nurse would reassess the pain and then call the health care provider to report
findings. Using SBAR (situation, background, assessment, recommendations), the nurse could ask for orders for
diagnostic testing or for a change in pain medication.
19.
20.
21.
22.
23.
24.
25.
26.
27.
28.
29.
30.
Answer 1: While waiting for the health care provider to call back, the nurse should enlist the unlicensed assistive
personnel to take and report vital signs. The other actions are correct, but the nurse is responsible for those tasks.
Answer 3: Blood thinners, aspirin, antiinflammatory medications and vitamin E are likely to be discontinued before
surgery.
Answer 4: Pain is likely to be severe due to tissue ischemia. The other symptoms could also occur.
Answer 1, 3, 5: Patients with sickle cell disease should avoid high altitudes, flying in unpressurized planes,
dehydration, extreme temperatures, iced liquids, alcohol, and vigorous exercise. Patients should not smoke and
should protect extremities from injury because of impaired circulation. Patients with sickle cell disease should get
vaccinated and take prophylactic antibiotics to protect against infections.
Answer 2: In polycythemia, the blood is very viscous and there is an increased risk of venous thrombosis. There is a
potential for life-threatening pulmonary emboli if the clot breaks off and travels to the lungs. The nurse would
perform all of the other assessments as part of total patient care.
Answer 2: For the patient’s safety and protection from infection, the nurse would initiate protective isolation, wash
hands, and don appropriate apparel (e.g., mask, gown, gloves), then check the patient for signs of infection. Hand
hygiene is important to stress to the patient, but it’s more important to inform visitors and all caregivers. The
medication list should be reviewed because adverse reactions to medication is the primary cause of agranulocytosis
(severe reduction of white cell components).
Answer 1: Drawing pictures and storytelling will help the child express fears and worries. The child is likely to need
protection against infection and be in protective isolation. Treatments include chemotherapy and bone marrow
transplant. In addition, the usual processes that combat infection are altered. Exposure to animals, plants, or other
people should be avoided during neutropenic episodes.
Answer 1, 2, 3, 4: Ecchymoses and petechiae suggest that the patient bruises very easily. This could be the result of a
coagulation disorder or a medication such as prednisone. The nurse asks questions to determine if the patient has
noticed bleeding from other sources. Asking the patient about the cause of bruises is also appropriate to identify
specific trauma or injury to the bruised areas. Hydrocortisone cream is not useful in this case. Dietary assessment is
always useful, but in this case is more related to the patient’s general health than to the specific finding of
ecchymoses and petechiae.
Answer 2: With a low platelet count, the nurse initiates bleeding precaution measures. Placing pressure on the arms
or legs during movement can cause bruising. A mask is not necessary, but good hand hygiene is always appropriate.
Patients with sickle cell disease would be encouraged to drink fluids to prevent dehydration. Patients with red
blood cell disorders are more prone to fatigue; however, the nurse would assess all patients for ability to achieve
activities of daily living and instruct the unlicensed assistive personnel accordingly.
Answer 4: Non-contact sports such as golf would be recommended because of the potential for injury in other
sports.
Answer 1: In the early stages, the patient may report a painless enlargement of a cervical, axillary, or inguinal lymph
node. Night sweats, weight loss, and fever are “B” symptoms associated with a poor prognosis. Alcohol-induced
pain is a feature associated with Hodgkin’s, but does not consistently manifest in every patient.
Answer 2, 3, 4: By the time non-Hodgkin’s is detected and diagnosed, the disease is usually widespread.
Involvement of the digestive organs is likely, but the lymph system could spread the disease and cause pressure in
any area. Pleural effusion, bone fractures, and paralysis are possible complications. Chemotherapy is the mainstay of
treatment for nonlocalized disease. The prognosis is worse than Hodgkin’s and the diagnostic testing and treatment
are rigorous, so it is likely that the patient and family will need support. Localized pain in the spine that increases
with movement is more associated with multiple myeloma. Total assistance for activities of daily living is not
anticipated until the end stage of the disease.
Critical Thinking Activities
Activity 1
31. a. Pernicious anemia
b. Vitamin B12 injections, folic acid supplements, iron supplements, possible transfusions
c. The treatment must be lifelong. Failure to maintain treatment will result in death.
Activity 2
32. a. Iron deficiency anemia
b. Female, menses, recent pregnancy, history of stomach surgery
c. Tachycardia, spoon-shaped fingernails, headache, burning tongue; desire to eat clay, starch, and ice
d. Administration of iron: enteric-coated or sustained-release capsules are well not absorbed; iron should be taken
about 2 hours before or after a meal with vitamin C (ascorbic acid) or orange juice to enhance iron absorption;
tetracycline antibiotics and quinolones (Cipro, Levaquin, Noroxin) should be administered within 2 hours of
iron; antacids interfere with iron absorption; if a dose is missed, continue with schedule; do not double a dose.
Dilute liquid iron preparations in juice or water; administer with a straw to avoid staining teeth. Provide oral
hygiene after taking.
Side effects: nausea, vomiting, constipation or diarrhea, and green to black stools. If side effects develop, the
dose and type of iron supplement may be adjusted.
Contraindications: Iron supplements may be contraindicated in peptic ulcer disease, sickle cell disease and
polycythemia.
Toxicity: Iron is toxic, and caution must be taken to store iron preparations out of a child’s reach.
Activity 3
33. a. Ambulation helps counter hypercalcemia because weight-bearing helps the bones reabsorb some calcium.
Calcium reabsorption in the bones decreases the risk of pathologic fractures. Fluids prevent dehydration and
dilute calcium and prevent protein precipitates that can cause renal tubular obstruction.
b. First, the nurse would assess the pattern of pain and plan activities for when pain is lower and energy is higher.
Medicate the patient 30-40 minutes before ambulation and explain the benefits of ambulation. Obtain assistive
devices as needed; for example, a wheelchair can be nearby if the patient wants to stop and rest. To increase
sense of control, encourage the patient to take an active role in the design of the ambulation program. Enlist the
family as appropriate. Setting small goals—for example, walking to the end of the hall—is also helpful.
Activity 4
a. Providing care under these circumstances may seem over whelming, but use the principles of: nursing process,
ABCs (airway, breathing, circulation), and delegation. Assess the situation: Can I accomplish everything by
myself? Is help immediately available? What is necessary but not immediately life-saving? Use ABCs: What
are the priorities of patient care? What are the priorities of the orders? What can and should be done quickly?
Use delegation: What can be delegated? What skills do you have? What skills do others have?
b. Assuming you are familiar with the equipment in the room, lowering the bed and giving the patient oxygen
are probably the quickest tasks to perform. Whoever (you, health care provider, RN) is best at venipunctures
should start the IV and give the fluid bolus. Pressure on the wound should be delegated to the unlicensed
assistive personnel (UAP). When the bleeding is slowed, apply the pressure bandage and direct the UAP to
take vital signs. Calling the blood bank, laboratory, and admissions office for an intensive care bed should be
delegated to the unit secretary. Typically, the blood products will not be immediately available (i.e., 15-60
minutes). The LPN/LVN should alert the RN, so that blood can be started immediately when it arrives on the
unit.
CHAPTER 8—CARE OF THE PATIENT WITH A CARDIOVASCULAR OR A PERIPHERAL VASCULAR DISORDER
Tracing a Drop of Blood
1.
Superior or inferior vena cava  right atrium  tricuspid valve  right ventricle  pulmonary semilunar valve 
pulmonary artery  capillaries in the lungs  pulmonary veins  left atrium  bicuspid valve  left ventricle 
aortic semilunar valve  aorta
Figure Labeling
2.
See Figure 8-1.
a. Anterior right atrial branch of right coronary artery
b. Right coronary artery
c. Marginal branch of right coronary artery
d.
e.
f.
g.
Anterior interventricular branch of left coronary artery
Marginal branch
Circumflex branch of left coronary artery
Left coronary artery
Table Activity
3.
Scale
Description of pulse
0
Absent
+1
Barely palpable, intermittent
+2
Weak, possibly thready, but constantly palpable and with consistent quality
+3
Normal strength and quality
+4
Bounding, easily palpable, may be visible
Short Answer
4.
Impulse pattern: SA node AV node  bundle of His  right and left bundle branches of AV bundle Purkinje
fibers
5. Nonpharmacologic therapies for hypertension includes: losing excess weight, exercising regularly, reducing saturated
fat and sodium, consuming sufficient potassium, calcium, and magnesium, limiting alcohol intake, smoking cessation,
and using relaxation techniques and stress management
6. The signs and symptoms of the classic five Ps of arterial occlusion include: pain, pulselessness, pallor, paresthesia, and
paralysis.
Multiple Choice
7. Answer 4: Troponin I is the most useful in diagnosing a myocardial infarction because it is cardiac muscle specific. The
other cardiac markers should also be reported; however, those values can be affected by skeletal muscle damage,
other muscle disorders or renal problems.
8. Answer 3: The patient would benefit from any of these topics; however, reducing fat in the diet and reducing weight
are the most important topics for lowering cholesterol levels. An overall serum cholesterol level of less than
200 mg/dL is desirable, 200 to 239 mg/dL is borderline high, and more than 239 mg/dL is high.
9. Answer 1: Causes of sinus tachycardia include exercise, anxiety, fever, shock, medications, heart failure, excessive
caffeine, recreational drugs, and tobacco use. In the other options, sinus bradycardia is more likely to be the
observed rhythm
10. Answer 4: Atrial fibrillation is a very rapid chaotic production of atrial impulses. Heart muscle contractions are too
weak to reach the periphery and this manifests as a pulse deficit. To assess for a pulse deficit, one person counts
the apical pulse while the other person counts a peripheral (radial) pulse. The deficit is the difference between the
two.
11. Answer 3: Premature ventricular contractions (PVCs) are early abnormal ventricular beats that occur in conjunction
with the underlying rhythm; therefore the nurse will note a change in the rate and rhythm of the palpated pulse.
Some patients are asymptomatic; others may experience palpitations, weakness, and lightheadedness.
12. Answer 2, 3, 5, 6: Signs and symptoms of cardiac arrest include abrupt loss of consciousness with no response to
stimuli, gasping respirations followed by apnea, absence of pulse (radial, carotid, femoral, and apical), absence of
blood pressure, pupil dilation, and pallor and cyanosis.
13. Answer 4: Apply oxygen first. The other interventions are also necessary for patients with STEMI MI (an acute
myocardial infarction caused by complete interruption of blood flow which causes ST elevation).
14. Answer 3: Severe pitting edema will progress upwards and it is likely that there is fluid retention throughout the
body. The nurse would first observe respiratory effort. If the patient needs immediate intervention for dyspnea or
orthopnea, the nurse would intervene (e.g., position change, oxygen and diuretics as ordered). The nurse will also
make the other assessments.
15. Answer 2: Diuretics will increase urine output and electrolyte imbalances can occur. In particular, potassium must
be monitored because hypokalemia or hyperkalemia can cause cardiac dysrhythmias.
16. Answer 4: Inoperable coronary artery disease is an indication for transplant. The other options represent
contraindications. See Box 8-7 for additional information.
17. Answer 1: Prothrombin time, International Normalized Ratio, and partial thromboplastin time reflect blood clotting,
so these laboratory values are the most important to follow up for patients who are on anticoagulant therapy. The
electrolytes are important for heart muscle contraction. Enzyme creatine kinase, creatine phosphokinase, and
myoglobin can be used to assist with the diagnosis of myocardial infarction, but troponin levels are now more
commonly used. B-type natriuretic peptide is used in the diagnosis of heart failure.
18. Answer 4: Low hemoglobin indicates decreased ability to carry oxygen to the body cells and anemia, so the first
action is to make sure that the patient is getting supplemental oxygen. (Oxygen is likely to have been previously
ordered for a diagnosis of myocardial infarction; if not, the nurse should start oxygen and then obtain an order.) The
other options could also be included to correct low hemoglobin.
19. Answer 1: During cardiac catheterization, the catheter is inserted into a peripheral vessel (usually the arm or the
groin). There is a potential for bleeding or injury to nerves; pulses and sensation distal to the site of insertion must
be checked. Electrocardiograms and positron emission tomography are considered noninvasive.
20. Answer 2: Smoking cessation or at least reducing the number of cigarettes is a modifiable factor. Heredity plays a
role but is considered nonmodifiable. Prophylactic drugs would not be the first line of therapy for this healthy
patient. Discussions of diet and exercise would be more appropriate. Body mass index of 30 is too high because this
indicates obesity.
21. Answer 1, 2, 3, 4: Blood glucose control, monitoring blood pressure, healthy eating and regular exercise are
recommendations for all patients, but persons with diabetes have a higher risk for cardiac problems. The nurse
would not recommend aspirin therapy, because prescribing medication is outside the nurse's scope of practice. The
nurse could suggest that the patient talk to the health care provider about aspirin therapy.
22. Answer 4: Recent studies indicate that type D personality has the highest risk for cardiovascular problems because
of increased anxiety and depression. The type A personality who is in a hurry and often angry or irritated was
formerly believed to have the highest risk.
23. Answer 2: The monitor is showing a normal sinus rhythm. (Note to student: If there is ever any doubt about the
monitor function or display or if you doubt your interpretation of the electrocardiogram tracing, just check on the
patient.)
24. Answer 4: Recall that bearing down is one way to cause vagal stimulation. The other options can also cause sinus
bradycardia but are less likely to have such a rapid recovery to a regular rate.
25. Answer 3: In third-degree heart block, the impulses to stimulate heart muscle contraction are not being transmitted
through the atrioventricular junction. The rate is very slow and hypotension and angina are likely.
26. Answer 1: For this patient, there is an increased risk for ventricular fibrillation. The patient may or may not have
symptoms during the episodes, but aggressive treatment is initiated to prevent ventricular fibrillation, which is a
lethal dysrhythmia. Beta-adrenergic blockers are used in the ongoing suppression of ventricular tachycardia.
27. Answer 2: Ventricular fibrillation can be reversed if an electrical countershock is applied using the defibrillator. If
defibrillation fails to convert the dysrhythmia, a bag-valve-mask with supplemental oxygen and a crash cart will be
needed. A temporary pacemaker is not typically used for ventricular fibrillation.
28. Answer 4: The arm on the pacemaker side should be immobilized for the first several hours; then for 6-8 weeks, the
patient must refrain from lifting the arm over the head. Climbing stairs and participation in active sports are more
related to recovery during cardiac rehabilitation. Electrical sources may interfere with the pacemaker’s fixed mode.
29. Answer 4: Stents are thrombogenic; thus, the patient is likely to be prescribed an anticoagulant.
30. Answer 2: Applying patches in the morning and removing them at bedtime prevents the development of tolerance.
Nitroglycerin tablets should always be available in a pocket or purse. A burning sensation under the tongue is
expected during activation of the tablet. Up to three tablets should be taken to determine if pain relief is adequate.
31. Answer 3: Pain is the foremost symptom and is the target of immediate therapy, because pain is a signal of ischemia.
32.
33.
34.
35.
36.
37.
38.
39.
40.
41.
42.
43.
44.
45.
46.
Diaphoresis is secondary to pain or possibly hypotension. Palpitations could occur, but are not a typical complaint.
Shortness of breath is related to the body’s attempt to increase oxygen to the tissues.
Answer 4: Fortunately, rheumatic fever now occurs less frequently in the United States, because treatment for group
A -hemolytic streptococci infections has improved. For older patients or for patients who have emigrated from
undeveloped countries, the possibility for rheumatic heart disease still exists.
Answer 3: First, the nurse would determine if the correct dose and form of the nitroglycerin were taken. If the
nitroglycerin was taken correctly, than the nurse may opt to quickly assess for other symptoms that suggest cardiac or
digestive problems. Based on the assessment, the nurse may decide to call 911 or the health care provider. The
neighbor should not drive himself to the hospital.
Answer 2: Thrombolytics are not used for patients with active internal bleeding, suspected aor tic dissecting
aneurysm, recent head trauma, history of hemorrhagic stroke within the past year, or surgery within the past 10
days.
Answer 4: For 24-48 hours, the patient is usually limited to getting up to the bedside commode; thereafter, the
activity is gradually increased, but the nurse should carefully assess the patient before and after exertion and then
give the unlicensed assistive personnel additional instructions about how to assist the patient.
Answer 4: Teaching him how to read the labels gives him a practical skill that he can use at the grocery store. The
other options are incorrect. Healthy fats that do not exceed 30% of the total calories are part of good nutrition. Fiber
intake should be 20-30 grams.
Answer: 2.27 rounded to 2.3 liters.
One liter of fluid equals 1 kg (2.2 pounds); a weight gain of 2.2 pounds signifies a gain of 1 liter of body fluid.
2.2 pounds : 5 pounds = 2.272
1 liter
x
Answer 1: The patient is describing a strategy that he uses to deal with orthopnea. Worsening heart failure is
accompanied by fluid retention and it is likely that sleeping in a chair is causing the fluid to collect in the lower
extremities. As the edema worsens, the abdominal girth will increase and the breathing will become more labored as
the fluid progresses upwards. The nurse is also likely to assess compliance with diet and medications. The home
health nurse has an additional advantage of being able to look at the environment. Climbing stairs or navigating
distances between rooms may be an issue as the patient becomes progressively more fatigued.
Answer 1: Sudden absence of pulse may indicate an arterial occlusion. Permanent tissue damage can occur if
blockage persists. Capillary refill can be slower than normal for patients with peripheral vascular disease.
Erythema and edema may accompany infection or venous thrombosus. A tingling or burning sensation (paresthesia)
should be investigated as an early sign of decreased tissue perfusion
Answer 2: Remember the priorities of airway and breathing and give the patient oxygen. Next establish a peripheral
IV for morphine and diuretics. Arterial blood gases and auscultating lung sounds will assist in the diagnosis of
pulmonary edema, but the patient is in severe distress and the symptoms are attended to first.
Answer 3: The unlicensed assistive personnel can weigh the patient. The other tasks are nursing responsibilities.
Answer 1: In pericarditis, the membranous sac that surrounds the heart becomes inflamed. Fluid collects in the sac
and the heart becomes compressed by the pressure of the fluid. The effusion restricts the movement of the heart
(cardiac tamponade).
Answer 3: Endocarditis puts the patient at risk for emboli that can travel to any organ. Sudden shortness of breath
suggest that a large embolus or numerous small emboli have lodged in the lungs. The other signs/symptoms are
part of the presenting clinical manifestations.
Answer 2: Prompt treatment with intensive antibiotic therapy will now cure a majority of the patients with
endocarditis. Valve replacement is considered an adjunct procedure. Complete bedrest is usually not ordered,
unless there is high fever or signs of heart failure. Anticoagulant therapy is not usually ordered because of risk for
intracranial bleeding.
Answer 3: Cardiomyopathy can be caused by cocaine abuse. Cocaine causes intense vasoconstriction of the coronary
arteries and peripheral vasoconstriction, resulting in hypertension. Cocaine also causes high circulating levels of
catecholamines, which may further damage myocardial cells, leading to ischemic or dilated cardiomyopathy. The
prognosis is poor. Excessive alcohol intake over a prolonged period of time also increases the risk.
Answer 1: Transplant patients need immunosuppressive therapy and protective isolation. Pericardiocentesis is
performed for cardiac tamponade. Percutaneous transluminal angioplasty is diagnostic and reparative for coronary
47.
48.
49.
50.
51.
artery disease or embolism.
Answer 4: The prehypertensive category (blood pressure of 120 to 139/80 to 89 mm Hg) was created to help people
recognize that small increases in blood pressure can have large consequences on health. Patients would be advised
about controlling modifiable risk factors and encouraged to participate in routine health appointments.
Answer 3: For arterial insufficiency, the leg should be dependent, because this will increase the blood flow to the
tissues and help decrease the pain. The other options are likely to increase pain. Elevation and ice will decrease the
blood flow. Exercise must be balanced with rest.
Answer 1: Dark-green vegetables contain vitamin K which counteracts the effect of the anticoagulant drug. The other
options are good dietary advice for most patients.
Answer 2: The patient is showing signs and symptoms of a ruptured aneurysm and hypovolemic shock. The nurse
would place the patient in a shock position and immediately call for help. (Note to student: See Chapter 7 nursing
interventions for hypovolemic shock for additional information. Rapid response team, code team, or hospitalist may
be available in different facilities.) The patient does need a patent IV. Giving pain medication is not a priority,
although oxygen should be started.
Answer 3: Early ambulation and encouraging mobility, which includes change of position and range-of-motion
exercises are the most important preventive measures. Compression stockings and calf measurements are part of
prevention and detection. Elevating the legs may be ordered as a comfort measure if venous thrombosis occurs.
Critical Thinking Activities
Activity 1
52. a. The nurse seeks subjective data: onset, location, quality, duration, radiation, and self-care of measures to relieve
the pain. Accompanying symptoms: shortness of breath, dizziness, weakness, anxiety, fear, or unusual fatigue.
Collection of objective data includes: apprehension and anxiety, hypotension, tachycardia or a barely perceptible
pulse, early temperature elevation, diaphoresis; vomiting, ashen color; cool, clammy skin; labored respirations;
and cardiac dysrhythmias. If possible, find out about risk factors. The nurse should take vital signs and check
pulse oximeter reading. Other physical assessments (e.g., auscultating heart and breath sounds) are deferred
until the chest pain and respiratory distress are under control.
b. Myocardial infarction
c. Prevention of further tissue damage, interventions to promote tissue perfusion
d. Administer oxygen as prescribed (oxygen relieves pain and helps to preserve cardiac muscle).
Administer prescribed analgesics (usually morphine sulfate, which relieves pain, reduces anxiety, causes
vasodilation of vascular smooth muscle, and reduces myocardial workload). Employ alternative methods of
pain relief measures.
Maintain bed rest with head of bed elevated 30 degrees for first 24-48 hr. Provide calm, restful environment;
increase activity level as prescribed.
Monitor and infuse IV feedings as ordered.
Administer prescribed medications such as antidysrhythmics, nitrates, and beta blockers.
Assess and monitor vital signs q 4 hr. Palpate for pedal pulses, assess capillary refill, auscultate heart and
breath sounds, assess for pedal or dependent edema q 4 hr. Monitor intake and output (I&O)
Activity 2
53. a. Native American, history of hypertension
b. Nitroglycerin, aspirin, beta-adrenergic blocking agents such as propranolol, metoprolol (Lopressor), nadolol
(Corgard), atenolol (Tenormin), and timolol (Blocadren); and calcium channel blockers such as nifedipine
(Procardia), verapamil, diltiazem, and nicardipine (Cardene). Ticlopidine (Ticlid) or clopidogrel (Plavix) may be
given for anti-platelet therapy.
Activity 3
54. a. Changes in the cardiac musculature lead to reduced efficiency and strength. This causes decreased cardiac
output, which can manifest as disorientation, syncope, and decreased perfusion to body tissues. Arterial
changes related to aging cause hypertension with increased cardiac effort needed to pump blood through the
circulatory system. Edema, secondary to heart failure, may cause tissue impairment in the immobile older adult.
Immobility leads to venous stasis, venous ulcers, and poor wound healing. It also increases the risk of venous
thrombosis and embolus formation. Polypharmacy and decreased drug metabolism and excretion contribute to
toxicity.
b. Signs and symptoms of heart failure include:
Decreased cardiac output
• Fatigue
• Anginal pain
• Anxiety
• Oliguria
• Decreased gastrointestinal motility
• Pale, cool skin
• Weight gain
• Restlessness
Left ventricular failure
• Dyspnea
• Paroxysmal nocturnal dyspnea
• Cough
• Frothy, blood-tinged sputum
• Orthopnea
• Pulmonary crackles (moist popping and crackling sounds heard most often at the end of inspiration)
• Radiographic evidence of pulmonary vascular congestion with pleural effusion
Right ventricular failure
• Distended jugular veins
• Anorexia, nausea, and abdominal distention
• Liver enlargement with right upper quadrant pain
• Ascites
• Edema in feet, ankles, sacrum; may progress up the legs into thighs, external genitalia, and lower trunk
c. Heart failure is managed with digoxin, vasodilators, Angiotensin-converting enzyme (ACE) inhibitors, beta
blockers, and angiotensin II receptor blockers. Nesiritide reduces pulmonary capillary pressure, helps improve
breathing, and causes vasodilation with increase in stroke volume and cardiac output.
d. Teach the patient to monitor for signs and symptoms of recurring problems such as shortness of breath; swelling
of ankles, feet, or abdomen; and frequent nighttime urination. Plan activity to provide for rest periods; take
medications as prescribed; report signs of nausea, pain, lightheadedness, and syncope to the doctor. Eat foods
high in potassium and low in sodium if taking diuretics. Avoid alcohol when taking vasodilators.
Activity 4
55. a. Venous stasis ulcers result from vein insufficiency causing stasis of blood. People who are homeless spend a lot
of time with their legs in a dependent position. This puts greater strain on vessels. The corrective measure is to
lie down and elevate legs, but this is not always possible for homeless persons. Poor nutrition, exposure to the
elements, and lack of access to hygienic facilities impairs healing of ulcers.
b. P for pulses: Assess the patient’s affected extremity first. Compare current findings with previous. Pulses
should be present in venous disorders, but edema may interfere with palpation. Use a Doppler as needed.
A for appearance: Note whether the extremity is pale; mottled; cyanotic; or discolored red, black, or brown.
T for temperature: If the problem is venous, the extremity will feel normal or abnormally warm.
C for capillary refill: Capillary refill is normally less than 2 seconds, but it may be extended when the patient
has peripheral vascular disease (PVD).
H for hardness: Palpate the extremity to determine whether the tissues are supple or hard and inelastic.
Hardness may indicate long-standing PVD, chronic venous insufficiency, lymphedema, or chronic edema.
Hardened subcutaneous skin also increases the risk of stasis ulcers.
E for edema: Pitting edema frequently indicates an acute process, and nonpitting edema may be seen with
chronic conditions, such as venous insufficiency.
S for sensation: Ask about pain, and other abnormal sensations, such as numbness or tingling, or heat or
cold.
c. Visibly ulcerated skin having dark pigmentation, dryness, scaling, and edema may occur. Dull aching pain
relived by elevation of the extremity. Peripheral pulses are usually present with venous insufficiency. Pain,
aching, and cramping associated with venous disorders are usually relieved by activity and/or elevating the
extremity.
d. The focus is on promotion of wound healing and preventing infection. Dietary management includes: adequate
protein intake with supplements of vitamin A and C, and mineral zinc. Wound treatment includes: débridement
of necrotic tissue, antibiotic therapy, and protection of ulcerated areas. Homeless patients may need assistance in
obtaining medication, wound treatments, or nutritious foods. The nurse should suggest ways to adapt wound
care and instruct about elevating legs whenever possible.
Activity 5
56. Recall the patient teaching points when you are doing the food product calculations.
• Recommended daily intake is 2 g sodium, 1500 calories, and low cholesterol.
• Limit total fat intake to 25% to 35% of total calories each day. Limit intake of saturated fats to less than 7% of
total fat intake. Teach the patient that saturated fats (e.g., shortening, lard, or butter) are solid at room
temperature; better sources of fat include vegetable, olive, and fish oils.
• Teach the patient to avoid foods high in sodium, saturated fats, and triglycerides. Review alternative ways of
seasoning foods to avoid cooking with salt. Explain the need to limit intake of eggs, cream, butter, and foods
high in animal fat. Teach the patient and family how to read labels on foods.
• Teach the patient to eat 20-30 g of soluble fiber every day. Foods such as bran, beans, and peas help lower bad
cholesterol (low-density lipoprotein).
Recommendations will be based on what you found on the shelf. Typically, canned foods are higher in sodium
than fresh foods and frozen premade meals are higher in fat. For elderly housebound people, canned or frozen food
is likely to be more convenient, but some product lines are better than others. One suggestion for single elders (or
busy nursing students) is to make a batch of healthy homemade soups, beans, casseroles, etc., and freeze in singleserving portions.
CHAPTER 9—CARE OF THE PATIENT WITH A RESPIRATORY DISORDER
Matching
1.
d
2.
e
3.
f
4.
b
5.
g
6.
h
7
a
8.
c
9.
j
10. i
Table Activity
11.
pH
7.35-7.45
PaCO2
35-45 mm Hg
PaO2
80-100 mm Hg
HCO3–
21-28 mEq/L
SaO2
95%
Multiple Choice
12. Answer 4: Eustachian tubes are on both sides of the nasopharynx and connect to the middle ear. The inner linings of
the pharynx and the eustachian tube are continuous, and an infection of the pharynx can spread easily to the ear.
This is common in children.
13. Answer 1: The epiglottis protects the larynx when swallowing; it covers the larynx tightly to prevent food from
entering the trachea (aspiration) and directs the food to the esophagus.
14. Answer 3: The expected respiratory rate is 40 to 60 breaths/minute for a newborn.
15. Answer 2: Medulla oblongata and pons of the brain (brain stem) are responsible for the basic rhythm and depth of
respiration. Uncontrolled and excessive intracranial pressure can cause brain herniation, which will result in
respiratory arrest.
16. Answer 4: These symptoms can manifest during an acute asthma episode, but retractions are considered a late-stage
of respiratory distress. The nurse would ask the parent if rescue medications were given. If symptoms continue and
fail to respond to usual treatment (status asthmaticus), the patient is at risk for respiratory failure.
17. Answer 2: If the person has an episode of epistaxis or reports frequent episodes, check his or her blood pressure.
Other causes include dryness, chronic infection, trauma (e.g., injury, vigorous nose blowing, or nose picking), topical
corticosteroid use, nasal spray abuse, street drug use, and disorders that prolong bleeding time or reduce platelet
counts. Aspirin, nonsteroidal antiinflammatory drugs, and anticoagulants can prolong bleeding times. If the
neighbor were having active bleeding, holding pressure is the best advice. If the neighbor gives additional
information that suggest problems with clotting (e.g., bleeding from other sites), an appointment for laboratory
work would be appropriate. If the neighbor reported an injury to nose/face, then the nasal septum should be
examined.
18. Answer 4: Progressive or persistent hoarseness is an early sign of laryngeal cancer. Signs of metastasis to other areas
include pain in the larynx radiating to the ear, difficulty in swallowing, a feeling of a lump in the throat, and
enlarged cervical lymph nodes.
19. Answer 1, 3, 4, 6: Pneumococcal vaccination (two types: pneumococcal polysaccharide vaccine and pneumococcal
conjugate vaccine) is recommended for all babies and children younger than 2 years old, all adults 65 years or older,
and people 2 through 64 years old with certain medical conditions (e.g., diabetes, chronic obstructive pulmonary
disease) and adults 19 through 64 years old who smoke cigarettes. For additional information see
http://www.immunize.org/askexperts/experts_pneumococcal_vaccines.asp
Occupational exposure and traveling are not currently considered risk factors for healthy adults.
20. Answer 2: If the patient has a pneumothorax, auscultation of the lungs will reveal bilaterally unequal breath sounds,
with no breath sounds over the affected area. See Table 9-1 for additional information about breath sounds.
21. Answer 3: Air cannot pass over the vocal cords, so normal speech is impossible. The patient can breathe through the
tracheostomy opening. Secretions will be produced, but interventions relate to keeping the skin around the opening
clean and dry. The esophagus and trachea do not communicate, so choking is not anticipated.
22. Answer 1, 2, 3, 5, 6: The nurse would ask the patient to describe symptoms, onset, alleviating factors, and changes in
ability to perform activities of daily living. Patients with chronic lung disorders can have abnormal blood gas results
but past results are less relevant to the current status.
23. Answer 4: Flaring of the nostrils is usually considered a late sign. Increased respiratory rate is associated with many
conditions. Some are serious (e.g., pulmonary edema), and others are benign (aerobic exercise). Adventitious breath
sounds can be present, and the patient may not be aware that there is a problem (e.g., immobile patients can have
crackles). The orthopneic position does signal respiratory distress but is also used by many patients who have
chronic respiratory disorders.
24. Answer 2: Trauma combined with uneven chest expansion are associated with pneumothorax (collapsed lung).
25. Answer 1: The advantage of the helical computed tomography scan is that the entire study can be performed in less
than 30 seconds. The disoriented patient may have difficulty cooperating for a V-Q scan or pulmonary angiography,
as both are much longer procedures. A flat plate of the abdomen is the best exam for ingested foreign bodies. A
26.
27.
28.
29.
30.
31.
32.
33.
34.
35.
36.
37.
38.
39.
40.
41.
mediastinoscopy will be performed to obtain lymph tissue. A chest x-ray will be performed for the patient exposed
to tuberculosis.
Answer 2: The unlicensed assistive personnel (UAP) can assist the patient to move and make position changes. The
other tasks are nursing responsibilities. (Note to student: The UAP could ordinarily be expected to watch for and
report seeing blood in specimens; however, some blood is an expected finding after biopsy and the nurse should do
the assessment to determine if bleeding is excessive.)
Answer 1: Positioning the patient upright will facilitate the drainage and removal of the fluid from the thoracic
cavity.
Answer 3: Usually no more than 1300 mL of fluid is removed at one time because there is a risk of intravascular
fluid shifting that will result in pulmonary edema. Because of the risk for pulmonary edema, the nurse is likely to
increase the frequency of assessment. Giving the patient extra fluid could worsen fluid shifting. If the purpose was
therapeutic, the fluid may or may not have been sent to the laboratory for analysis.
Answer 4: Warfarin is an anticoagulant, so the nurse would hold pressure on the puncture wound for 20 minutes to
prevent a hematoma.
Answer 2: The student remembers that the automatic blood pressure cuff occludes blood flow to the distal portions of
the extremity, so the first pulse oximeter reading is likely to be falsely low.
Answer 3: With epistaxis, frequent swallowing suggests that the blood is running down the back of the throat. This
could either be rebleeding or posterior bleeding. Posterior bleeding is not always resolved with anterior packing.
Answer 1, 2, 3, 6: The goal is to keep the nasal mucous membranes moist, so a vaporizer, saline nose drops and
lubricants are recommended. Nose picking and putting other objects into the nose should be avoided; this point is
emphasized with pediatric patients. Aspirin is considered an anticoagulant. Blowing vigorously can restart
bleeding. (Note to student: The health care provider may have had the patient blow vigorously just prior to
examination, so the patient may assume that the action is okay.)
Answer 1: The nurse can administer the allergens and should mark the sites. The localized reaction should be
measured and documented. The health care provider is responsible for evaluating the outcomes of the test,
discussing allergens to avoid, and instructing the patient about ambiguous results. The nurse can reinforce what the
health care provider tells the patient.
Answer 3: The universal sign for choking is hand over the throat. People who are vigorously coughing should be
encouraged to continue coughing. While running out of the room is not an obvious signal, people have been known
to leave out of embarrassment. Waving hands frantically is a signal, but cause would have to be assessed.
Answer 3: Resting the voice is the most important measure to reduce the inflammation of the vocal cords. The other
measures help to promote comfort. Antibiotics are not prescribed for a diagnosis of viral laryngitis.
Answer 2: A rapid strep test is performed to detect the presence of -hemolytic streptococci, which is a severe form
of acute pharyngitis. If those results are negative, then the second swab is used to culture a medium and is allowed to
grow so the infecting organism can be identified.
Answer 3: The patient has symptoms of sinusitis. Transillumination involves shining a light in the mouth with the
lips closed around it; infected sinuses will look dark, whereas normal sinuses will transilluminate.
Answer 4: Dairy products thicken secretions, so they become more tenacious and harder to expectorate. Coffee,
soda and orange juice are not the ideal choices, if the patient has comorbid health conditions (e.g., caffeinated drinks
and sodas contribute to loss of calcium and orange juice is not recommended during urinary tract infections). Water,
diluted juices and electrolyte solutions are better choices.
Answer 4: The symptoms will mimic other respiratory disorders (e.g., influenza); thus, diagnosis is delayed because
more common causes will be investigated first. During this delay, the infection will become more entrenched.
Legionnaires’ and severe acute respiratory syndrome (SARS) can be transmitted via droplets in air, so many people
could be exposed before the diagnosis is made. Anthrax has been identified as a possible bioterrorism agent.
Morbidity is high for all three disorders. For Legionnaires’ disease, 1 in 10 will die. For SARS, 10-20% require
intubation and risk for death is high. Anthrax responds to antibiotics once the diagnosis is made.
Answer 4: The drug regimen is prolonged and for various reasons, many will fail to complete the therapy. This has
contributed to multidrug-resistant tuberculosis (TB) strains. Family and friends are generally not at high risk for
contracting TB. Hand hygiene and covering the mouth while coughing are encouraged as the main infection control
measures.
Answer 2: Severe pain in peripheral lung cancer is likely to be caused by a pleural effusion. The treatment for this is
a thoracentesis.
42. Answer 3: A pleural friction rub is considered diagnostic for pleurisy. The nurse should hear a dry, creaking,
grating, low-pitched sound with a machinelike quality during both inspiration and expiration. Crackles are
interrupted crackling or bubbling sounds more common on inspiration (e.g., pneumonia). Sonorous wheezes are
deep, loud, low, coarse sounds (like a snore) during inspiration or expiration (e.g., secretions, tumor, or spasm).
Sibilant wheezes are high-pitched, musical, whistlelike sounds during inspiration or expiration (e.g., foreign matter
or tumor compression).
43. Answer 4: Acetylcysteine (Mucomyst) is used to reduce the viscosity of secretions. This makes expectoration easier
and more effective.
44. Answer 1: The UAP can help the patient ambulate, but the nurse must give specific instructions about holding the
container below the chest and ensure that the UAP and patient do not place undue pressure on the tubes.
45. Answer 3: IV fluids are usually withheld to prevent adding fluids to the overloaded patient. (An IV saline lock
would be the expected order.) The other orders are appropriate for patients with pulmonary edema.
46. Answer 2: The nurse would first check vital signs and a pulse oximeter reading and assess for other signs of
respiratory distress or decreased cardiac output. Notifying the RN and health care provider would be the next step.
A blood gas is likely to be ordered. Assessing the leg is not helpful once the thrombus becomes an embolus.
47. Answer 1: Sepsis is the most common precursor of acute respiratory distress syndrome (ARDS). The window is 5-10
days after onset of sepsis. The mortality rate is 55% to 70% when ARDS is associated with sepsis. ARDS due to
injury usually manifests in 12-24 hours. Chronic obstructive pulmonary disease (COPD) or asthma can be
underlying factors, but many patients who have COPD or asthma never develop ARDS.
48. Answer 1: Care should be divided into short sessions with intermittent periods of rest. Hygienic care should not be
completely deferred; the nurse should determine how the care can be abbreviated or adapted and inform the UAP
accordingly. The nurse must assess the patient’s response to ambulation and patient’s ability to participate in rangeof-motion exercises and then inform the UAP.
49. Answer 4: An increased number of red blood cells (polycythemia) occurs as the body attempts to increase the
oxygen to tissue. Dehydration could contribute to an elevated red cell count, but is not directly related to chronic
bronchitis.
50. Answer 3: For newly diagnosed asthma patients, identification of allergens in the home environment will help them
to control/avoid exposure and will decrease episodes of acute attacks. These patients should be able to resume
normal activities after treatment for an acute episode.
51. Answer 1: The primary symptom of emphysema is dyspnea on exertion, which becomes progressively more severe.
The other findings will manifest as the disease progresses.
Critical Thinking Activities
Activity 1
52. a. Obstructive sleep apnea
b. Risk factors include obesity and male gender. Personal history factors include recent motor vehicle accident
caused by falling asleep and reports of loud snoring at night.
c. Mild sleep apnea can be corrected by avoiding sedatives and alcohol for 3-4 hours before sleep. Other corrective
measures include weight loss, use of oral appliances to bring the mandible and tongue forward to enlarge the
airway space, and support groups. In severe cases, nasal continuous positive airway pressure (nCPAP) may be
used. Bilevel positive airway pressure (BiPAP), delivers higher pressure during inspiration (when the airway is
most likely to be occluded) and lower pressure during expiration. BiPAP may be better tolerated.
Activity 2
53. a. Symptoms are generally mild. They may include cold symptoms, headache, anorexia, myalgia, and irritating
cough that produces mucopurulent or bloody sputum.
b. Blood and sputum cultures, chest radiographic studies, complete blood cell count, pulmonary function tests,
arterial blood gases (ABGs), and pulse oximetry
c. There is no definitive treatment for viral pneumonia. Medications that may be prescribed include analgesics,
antipyretics, expectorants, and bronchodilators.
d. Assessments should include vital signs, pulse oximeter readings, breath sounds, respiratory effort, assess
characteristics of sputum, tolerance of activities and mobility.
Activity 3
54. a. Drug therapy for tuberculosis (TB) lasts between 6 and 9 months and many people will begin to forget to take
medication once the symptoms are resolved. There is also a higher incidence of TB among older people, urban poor,
minority groups, immigrants, and the homeless. The barriers to care include financial concerns, access to facilities,
problems understanding the provider’s instructions, difficulty with follow-up care, and differences in health values
and beliefs.
b. First the nurse should seek the patient’s opinion on what would help increase compliance and the major
stumbling blocks in meeting that goal. Compliance can be increased for some by including family members in the
teaching sessions. For others, directly observed therapy allows a health worker to observe while the person takes the
medication. Education regarding the dangers of multidrug-resistant strains will encourage some. Others may need
help from social services to locate financial resources. Helping the patient link the medication to a routine activity
(i.e., brushing teeth) could help. An electronic reminder could be used.
Activity 4
55. a. Assessment should include:
• Breath sounds, vital sounds, pulse oximeter readings, respiratory effort, pain, patient's tolerance of the
therapy, insertion site and dressing
• Note the amount and characteristics of the drainage
• Monitor laboratory results—specifically ABGs, white blood cell count
• Observe for bubbling or fluctuations in the drainage bottle
b. Keep tubing as straight as possible. Keep all connections tight and taped at connections. Never elevate the
drainage collection receptacles above the level of the chest.
c. The absence of bubbling in the water seal chamber indicates possible occlusion of the system.
d. Bubbling should be intermittent. Constant bubbling indicates a leak in the system.
CHAPTER 10—CARE OF THE PATIENT WITH A URINARY DISORDER
Short Answers
1.
a. Controlling body fluid levels by selectively removing or retaining water
b. Assisting with the regulation of pH
c. Removing toxic waste from the blood
2.
a. Filtration of water and blood products occurs in the glomerulus of Bowman’s capsule.
b. Reabsorption of water, glucose, and necessary ions back into the blood occurs primarily in the proximal
convoluted tubules, Henle’s loop, and the distal convoluted tubules.
c. Secretion of certain ions, nitrogenous waste products, and drugs occurs primarily in the distal convoluted
tubule. This process is the reverse of reabsorption; the substances move from the blood to the filtrate.
3.
Urinary frequency, urgency, nocturia, retention, and incontinence are common with aging. These occur because of
weakened musculature in the bladder and urethra, diminished neurologic sensation combined with decreased bladder
capacity, and the effects of medications such as diuretics.
• Urinary incontinence can lead to a loss of self-esteem and result in decreased participation in social activities.
• Older women are at risk for stress incontinence because of hormonal changes and weakened pelvic
musculature.
• Older men are at risk for urinary retention because of prostatic hypertrophy.
• Urinary tract infections in older adults are often associated with invasive procedures such as catheterization,
diabetes mellitus, and neurologic disorders.
• Inadequate fluid intake, immobility, and conditions that lead to urinary stasis increase the risk of infection in the
older adult.
• Frequent toileting and meticulous skin care can reduce the risk of skin impairment secondary to urinary
incontinence.
4. See Box 10-1.
Table Activity
5.
Urinalysis
Constituent
Normal Range
Influencing Factors
Color
Pale yellow to amber
Diabetes insipidus, biliary obstruction, medications,
diet
Turbidity
Clear to slightly cloudy
Phosphates, white blood cells, bacteria
Odor
Mildly aromatic
Medication, bacteria, diet
pH
4.6-8
Stale specimen, food intake, infection, homeostatic
imbalance
Specific gravity
1.003-1.030
State of hydration, medications
Glucose
Negative
Diabetes mellitus, medications, diet
Protein
Negative
Renal disease, muscle exertion, dehydration
Bilirubin
Negative
Liver disease with obstruction or damage, medications
Hemoglobin
Negative
Trauma, renal disease
Ketones
Negative
Diabetes mellitus, diet, medications
Red blood cells
Up to 2 LPF
Renal or bladder disease, trauma, medications
White blood cells
0-4 LPF
Renal disease, urinary tract infection
Casts
Rare
Renal disease
Bacteria
Negative
Urinary tract infection
Figure Labeling
6.
See Figure 10-13.
Multiple Choice
7. Answer 2: Blood urea nitrogen (BUN) is a laboratory test used to determine the kidney's ability to rid the blood of
waste and urea. An elevated BUN can be related to renal disease, dehydration, diet high in protein or heart disease.
In a young healthy patient, the nurse would first ask about noncompliance with the NPO (nothing by mouth) for the
8 hours prior to the blood sampling.
8. Answer 2: In the oliguric phase, BUN and serum creatinine levels rise while urinary output decreases to less than
400 mL/24 hours. In the diuretic phase, blood chemistry levels begin to return to normal and urinary output increases
to 1 to 2 L/24 hours. Return to normal or near-normal function occurs in the recovery phase. Recovery begins as the
glomerular filtration rate rises. End stage renal disease is confirmed by elevated BUN of at least 50 mg/dL and
serum creatinine levels greater than 5 mg/dL.
9. Answer 4: In voiding cystourethrography, an indwelling catheter is inserted into the urinary bladder, and dye is
injected to outline the lower urinary tract. Radiographs are taken, and the catheter is then removed. The patient is
asked to void while radiographs are being taken.
10. Answer 3: A local anesthetic is administered, and a scope is inserted into the urethra. Blood tinged urine at the first
voiding can occur because of mechanical trauma from the procedure. The other findings are not related to
cystoscopy.
11. Answer 1: Pregnant health care staff should refrain from caring for patients who received radioactive substances.
12. Answer 3: For patients with urinary incontinence, secondary to neurologic disorder, urodynamic studies are use to
evaluate the activity level of the urinary bladder muscle. Rectal electromyography is an associated test, which
evaluates urethral pressures.
13. Answer 1, 2, 3, 5: Flank or abdominal pain are not anticipated with diuretic therapy or acute renal failure. The other
interventions are correct.
14. Answer 3: See Box 10-2 for additional information
15. Answer 1, 2, 4, 5: The nurse should assess the catheter entry site for signs of trauma or infection. The other
instructions are correct and these actions can be delegated to the UAP.
16. Answer 2: Sulfamethoxazole-trimethoprim (Bactrim) is an antibiotic used to treat infections, such as uncomplicated
urinary tract infections and urethritis. Completing antibiotics as prescribed is the most important information to
prevent reoccurrence and resistant strains of bacteria. Drinking at least 2000 mL is also recommended.
Phenazopyridine hydrochloride (Pyridium) will turn the urine orange. Diuretics are usually taken in the morning.
17. Answer 1: Ketones appear in the urine as the body converts fats into energy, because glucose is not available to use
as an energy source.
18. Answer 3: WBC casts in the urine indicate involvement of the renal parenchyma in renal disorders, such as acute
pyelonephritis or acute glomerulonephritis.
19. Answer 3: The normal range of specific gravity is 1.003-1.030. Diabetes insipidus is an endocrine disorder in which
the kidneys are unable to conserve water, so the urine is very dilute. The other three conditions will cause
dehydration and the specific gravity will increase.
20. Answer 2, 3, 4, 5: The serum creatinine test is used to diagnose impaired kidney function. With normal renal
excretory function, the serum creatinine level should remain constant and normal. Prostatitis could cause an
obstruction to flow, but the kidneys continue to produce urine normally.
21. Answer 1, 2, 3, 4: The normal range is less than 4 ng/mL. Elevated levels may result from prostate cancer,
inflammation or infection, urinary tract infection, or recent cystoscopy or prostatic biopsy.
22. Answer 3: For renal angiography, the nurse must assess circulatory status of the involved extremity every 15 minutes
for 1 hour, then every 2 hours for 24 hours. A kidney-ureter-bladder radiography and ultrasonography do not require
any special postprocedural care. For the intravenous pyelogram, the patient needs to be encouraged to drink water to
flush the dye from the system, and the venipuncture site should be routinely observed.
23. Answer 3: Cholinergic and anticholinergic medications may be administered during urodynamic studies to
determine their effects on bladder function.
24. Answer 4: Bedrest is instituted for 24 hours after the procedure. Mobility is restricted to bathroom privileges for the
next 24 hours, and gradual resumption of activities is allowed after 48-72 hours.
25. Answer 4: Osmotic diuretics are used for acute renal failure to prevent irreversible failure, but they are
contraindicated in advanced end-stage renal failure.
26. Answer 2: The patient with urge and functional incontinence will benefit the most from having an external condom,
because he is unable to get to the bathroom in time. The patient with Alzheimer’s is likely to pull the external
catheter off. If the patient with a urinary tract infection has problems with incontinence, antibiotic therapy should
resolve the problem. An enlarged prostate prevents flow, so the external catheter does not address the underlying
problem.
27. Answer 3: Painless hematuria is the primary sign. Other symptoms include changes in voiding patterns, signs of
urinary obstruction, or renal failure, depending on the extent of the disease process.
28. Answer 2: Spironolactone (Aldactone) is a potassium-sparing diuretic, so it is contraindicated for patients who have
hyperkalemia.
29. Answer 2: The nurse would advise the patient that diphenhydramine (Benadryl) can cause urinary retention. This
could add problems with passing urine, because BPH can cause an obstruction of urine flow. In addition, the nurse
would remind the patient that all OTC medications should be reviewed with the health care provider and on file
with the local pharmacist.
30. Answer 1: Kegel exercises are recommended in prevention and treatment of stress incontinence, which is loss of
urine during coughing, laughing, sneezing, or straining.
Kegel exercises are recommended for all patients who are able to practice conscious motor control over the
pelvic musculature to reduce present or future episodes of incontinence. Some patients who have Parkinson’s or
Alzheimer’s may be able to learn Kegel exercises, depending on cognition and motor control.
31.
32.
33.
34.
35.
Answer 3: The indwelling catheter is inserted to splint and support the suture line after reconstruction of the
urethra; thus, tension on the catheter could result in disruption of the surgical site. The other patients have catheters
primarily for drainage purposes.
Answer 1: In nephrotic syndrome, excess fluid in the body is the most common sign. Patients who develop acute
glomerulonephritis may report a preceding episode of sore throat or skin infection with fever and malaise. Burning
with urination, low-back pain, hematuria, and fever are more associated with cystitis. Dysuria, weak stream, and
increasing pain with bladder distention are seen in patients with urethral strictures.
Answer 4: Excess fluid causes edema and hypertension, so the patient is placed on bedrest until those symptoms
resolve. The patient is also likely to have orthopnea, so the head of the bed should be elevated.
Answer 2: Albumin and blood in the urine are early indicators of chronic glomerulonephritis. Residual urine is a
bladder outflow problem that is not related to actual kidney function. Retained urine in the bladder is suspected to
contribute to bladder cancer. Ketones in the urine are usually associated with diabetes mellitus, although diet and
medication could be factors. Prostate-specific antigen is a screening test for prostate cancer.
Answer 1: A nurse should never access the fistula to draw blood, to give fluids or to check patency, unless he/she
has had special training in dialysis procedures. Auscultating the arteriovenous fistula for bruit (adventitious sound
of venous or arterial origin heard on auscultation) and palpating arteriovenous fistula for thrill (abnormal tremor)
are correct. Checking pulses and assessing pain are part of the assessment that is performed after the patient returns
from dialysis, but are less directly related to the function of the fistula.
Critical Thinking Activities
Activity 1
36. a. Signs and symptoms include pain in the costovertebral angle, elevated temperature, chills, and pus in the urine.
b. Urinalysis: pus, bacteria, and leukocytosis present
Intravenous pyelogram: presence of an obstruction or degenerative changes
Activity 2
37. a. Urolithiasis
b. Ideally, the stone will be passed without intervention. Fluid intake should be increased and monitored. The
urine will be strained to check for the stone or “graveling.” Cystoscopy, surgical incision, or chemolytic
medications to dissolve the stone may be prescribed. Extracorporeal shock wave lithotripsy is an alternative to
surgery.
c. Dietary modifications to reduce the level of calcium phosphorus and purine-containing foods may be indicated.
These foods include cheese, greens, whole grains, carbonated drinks, nuts, chocolate, shellfish, and organ meat.
Fluid intake of at least 2000 mL/day is also recommended. Drug therapy depends on stone composition. Sodium
cellulose phosphate is given to prevent calcium stone formation. Aluminum hydroxide gel binds with excess
phosphorus and allopurinol (Zyloprim) reduces serum urate levels.
Activity 3
38. a. The patient may experience anorexia, nausea, vomiting, and edema. Special attention should be paid to signs of
hydration, including mucous membranes, skin turgor, and urine output. There may also be signs of drowsiness,
muscle twitching, and seizures.
b. In the oliguric phase, BUN and serum creatinine levels rise while urinary output decreases to less than 20 mL/hr
(less than 400 mL/24 hr). The oliguric phase may last from several days to weeks to months. Some patients may
experience the nonoliguric form, usually caused by nephrotoxic antibiotics, in which urinary output may exceed
2 L/24 hr. In the diuretic phase, blood chemistry levels begin to return to normal and urinary output increases to
1-2 L/24 hr. The diuretic phase usually lasts 1-3 weeks. Return to normal or near-normal function occurs in the
recovery phase. Recovery begins as the glomerular filtration rate rises. Recovery can take up to 1 year.
c. The wife should be advised this would not be the best option. The diet should be low in protein, potassium, and
sodium. Carbohydrates should be high. The items she is proposing to bring in are high in protein and sodium.
Activity 4
39. a. Women are more susceptible to urinary tract infections (UTIs) than men because the urethra is short and
proximal to the vagina and rectum.
b. Complaints may also include frequency, urgency, and nocturia. Abdominal palpation may also cause
discomfort over the bladder.
c. Antibiotics and urinary antiseptics
d. Teach the woman to cleanse the perineal area from front to back to prevent contamination of pathogens
(especially E. coli) from the rectum to the short urethra.
• Encourage drinking 2000 mL of liquids per day unless contraindicated.
• Instruct the patient to take all the prescribed medications, even though symptoms may subside quickly.
• Empty bladder as soon after intercourse as possible. If UTIs are associated with intercourse, recommend
cleansing of genitalia with soap and water prior to having sexual relations.
• Shower instead of tub baths.
• Limit use of bubble baths.
• Instruct the patient about early detection and testing with Chemstrip LN.
CHAPTER 11—CARE OF THE PATIENT WITH AN ENDOCRINE DISORDER
Matching
1.
b
2.
a
3.
d
4.
c
5.
g
6.
h
7.
e
8.
f
9.
k
10. l
11. i
12. j
Figure Labeling
13. See Figure 11-1.
14. Table activity (See Table 11-5 for additional information.)
Type of Insulin
Injection Time
(Before Meal)
Risk Time for Hypoglycemic
Reaction
Onset of
Action
Duration
Lispro (Humalog)
5-15 min
No meal within 30 min
15-30 min
3-4 hr
Regular
Humulin R
Novolin R
ReliOn R
30 min
Delayed meal or 3-4 hr after
injection
30-60 min
6-8 hr
NPH/Regular Mix
70/30
Humulin Mix 70/30
30-60 min
Delayed meal or 3-4 hr after
injection
30-60 min
18-24 hr
Glargine (Lantus)
Usually taken
at 9 PM, once
daily
Starting dose should be 20% less 1-2 hr
than total daily dose of NPH
24 hr
Multiple Choice
15. Answer 2: Hyponatremia triggers the earliest symptoms. Most signs and symptoms (nausea, vomiting, irritability,
confusion, tremors, seizures, stupor, coma, and pathologic reflexes) appear when serum sodium levels fall below
125 mEq/L.
16. Answer 1, 2, 3, 5: The nurse should assess the patient's activity cycle and then inform the UAP, so that care and
assistance does not interrupt the cycle. The other instructions contain sufficient detail and these tasks are within the
scope of practice of the UAP.
17. Answer 4: Weight loss may correct the problem. Exercise helps to improve metabolic control. Oral hypoglycemics
are added if lifestyle modification is not successful. Insulin is sometimes used for type 2 diabetics (especially during
hospitalization or after surgery). In type 2 diabetes, there is less risk for diabetic ketoacidosis, but stress and illness
will still have an impact.
18. Answer 4: Long term complications of diabetes mellitus includes: blindness, renal failure, amputation, heart disease,
hypertension and stroke.
19. Answer 2: Patients with Addison's disease should carry emergency hydrocortisone in case of addisonian crisis,
manifested by: sudden, severe drop in blood pressure; nausea and vomiting; an extremely high temperature; and
cyanosis, progressing to vasomotor collapse and possibly death. In pheochromocytoma, the primary concern is
extremely elevated blood pressure. Patients with type 1 diabetes are advised to carry a source of glucose (e.g., hard
candy or dextrose tablets). Thyroid storm is a life- threatening exaggeration of hyperthyroidism; goals of treatment
are to prevent cardiac collapse and hyperthermia.
20. Answer 4: Regular insulin is the only kind that can be given intravenously, so the nurse would call the provider for
clarification and also ask about parameters for blood glucose titration. (Note to student: In diabetic ketoacidosis,
the goal is to gradually decrease the blood glucose, so the nurse would not assume that the normal range of 60-99
mg/dL was the desired parameter.)
21. Answer 1, 2, 3: For hypothyroidism, early signs include weight gain, difficulty concentrating, constipation, and
fluid/weight gain. Late signs include mood swings, infertility (in women), acute fatigue syndrome, and depression.
22.
23.
24.
25.
26.
27.
28.
29.
30.
Answer 3: First, the nurse acknowledges the underlying feelings of change and loss. Option 1 is false reassurance.
Option 2 is a platitude. Option 4 may be a possibility after assessment, treatment, and discussion.
Answer 2: A school nurse would notify the parents, so the child could be evaluated by a health care provider (for
diagnostic testing to rule out giantism). A nurse who works with/for the health care provider would perform the
other options. The health care provider might also contact the school nurse and ask for regular height and weight
reports.
Answer 1, 2, 4: Nursing assessment and intervention for patients with diabetes insipidus is focused on fluid loss and
dehydration. Fluids should not be restricted. Patients should be assisted to ambulate because of fatigue. It is likely
that they are frequently walking to the bathroom during the day and at night; thus, encouraging additional
ambulation is not necessary.
Answer 1: For any of these patients, the nurse would be aware of the possibility of developing SIADH; however,
malignancies are the most common cause of SIADH; cancerous cells are capable of producing, storing, and releasing
ADH.
Answer 4: Brain edema will result in a change in mental status, progressive lethargy, or changes in personality.
These symptoms are followed by seizures and loss of deep tendon reflexes.
Answer 3: All of the findings are positive; however, a gradual increase of serum sodium is the purpose of the
therapy.
Answer 3: In the postsurgical period, patients who have had thyroidectomy surgery are encouraged to deepbreathe, but the nurse would check with the health care provider about coughing, because of potential strain on the
suture line.
Answer 1: The symptoms of hyperthyroidism, reflect an increased metabolism. Intolerance to cold, constipation, and
lethargy are symptoms of hypothyroidism. Skeletal pain, pain on weight-bearing, and paranoia are seen in
hyperparathyroidism. Polyphagia, polydipsia, and polyuria are characteristics of type 1 diabetes mellitus.
Answer 4: Levothyroxine (Synthroid) is a replacement therapy for patients with hypothyroidism; thus,
normalization of the level of thyroid-stimulating hormone indicates that the therapy is working. Normalization of
urine specific gravity would be a therapeutic goal for diabetes insipidus. Gradual improvement of serum sodium is
31.
32.
33.
34.
35.
36.
37.
38.
39.
40.
41.
42.
43.
44.
45.
46.
the treatment goal for SIADH. A blood glucose of 250 mg/dL is used as a target to initiate intravenous dextrose
solutions for patients who are being treated for diabetic ketoacidosis.
Answer 3: The three goals of thyroid crisis management are (1) to induce a normal thyroid state, (2) prevent
cardiovascular collapse, and (3) prevent excessive hyperthermia. In SIADH, the goals of therapy are to correct low
sodium levels and achieve fluid balance. In severe hypoglycemia, patients will have confusion and change of
mental status; restoring the glucose level to 60 to 99 mg/dL will restore mental status. In addisonian crisis, highdose hydrocortisone replacement therapy, restores cortisol and IV fluid is given for hypotension
Answer 3: Upon finding a palpable nodule, the health care provider would order diagnostic testing to rule out
thyroid cancer. Severe hypothyroidism in adults is called myxedema. It is characterized by edema of the hands, the
face, the feet, and periorbital tissues. Congenital hypothyroidism is called cretinism. Colloid goiter could manifest as
an unsightly enlargement of the thyroid gland or with dysphagia, hoarseness, or dyspnea.
Answer 1: Although the nurse may see that the patient would benefit from a MyPlate review, the dietary restriction
related to the hyperparathyroidism is dairy products.
Answer 3: Hyperparathyroidism causes an increase in serum calcium and the goal is to rid the body of the excess.
Thiazide diuretics are not used because they decrease renal excretion of calcium and thus increase the hypercalcemic
state. Diuretics can be used in acute renal failure to preserve kidney function or in disorders that cause fluid
retention, such as congestive heart failure. Diuretics are usually included in the regimen for hypertension.
Answer 2: In this emergency situation, the LPN/LVN recognizes that IV calcium can precipitate hypotension, serious
cardiac dysrhythmias, or cardiac arrest. Thus electrocardiographic monitoring is indicated when administering
calcium. Assessing for allergies, verifying medication prescriptions, and checking patency of the site are
responsibilities of the nurse who is administering the drug. (Note to student: When patients become unstable or
critical, the LPN/LVN should notify the health care provider and RN and the RN should assume care and
responsibility for the patient. The LVN/LPN uses knowledge and skills during a crisis to contribute to care of
patients under the supervision of the RN.)
Answer 2: Foods that are low in phosphorus are encouraged because calcium and phosphorus levels are reciprocal.
In other words, if the serum phosphorus level is lower, the calcium level will increase, which is desirable for these
patients.
Answer 2: Diabetes insipidus causes production of urine with a very low (dilute) specific gravity. The normal range
of specific gravity is 1.003 to 1.030.
Answer 3: Simple goiter is usually caused by a dietary insufficiency of iodine.
Answer 3: Cortisol is a glucocorticoid that provides extra reserve energy in times of stress. Aldosterone, the
principal mineralocorticoid, regulates sodium and potassium levels by affecting the renal tubules. Glucagon is a
pancreatic hormone, which responds to decreased levels of glucose in the blood.
Answer 4: Regular insulin is given via the intravenous route for hyperglycemia.
Answer 2: Corticosteroids should never be abruptly discontinued because of the risk inducing adrenal insufficiency.
The other options could be done under the supervision of the health care provider.
Answer 1: The skin is very thin and fragile and easily torn; thus, gentle handling is necessary. The nurse must assess
the skin; this cannot be delegated. Frequent washing or shaving could contribute to skin damage.
Answer 3: These are signs of impending addisonian crisis, which is potentially life-threatening and the health care
provider should be notified immediately. The frequency of assessment will increase because of acuity.
Documentation is always appropriate, but the patient’s condition must be addressed first.
Answer 2: Recall that epinephrine and norepinephrine are involved in the fight or flight response. Lethargy,
constipation, and depression could be evident in many disorders; however, hypothyroidism could cause these
symptoms. Kussmaul’s respiration, hypotension, and drowsiness are seen in patients with diabetic ketoacidosis.
Excessive thirst, increased urine output, and lethargy are seen in diabetes insipidus.
Answer 4: The glycosylated hemoglobin (HbA1c) blood test measures the amount of glucose that has become
incorporated into the hemoglobin within an erythrocyte; these levels are reported as a percentage of the total
hemoglobin. Because glycosylation occurs constantly during the 120-day life span of the erythrocyte, this test reveals
the effectiveness of diabetes therapy for the preceding 8-12 weeks. The other tests give limited results related to
current status.
Answer 3: Type 1 diabetics have the greatest risk for diabetic ketoacidosis, which can be brought on by minor illness.
Presence of ketones should be reported to health care provider.
47.
48.
Answer: 15 mL/hour
100 units : 3 units = 15 mL/hour
500mL
x mL
Answer 2: This patient is to have nothing by mouth for the procedure, so the nurse decides not to feed this conscious
patient, but to use the emergency protocol to administer 50% dextrose. Once the patient has received the bolus, the
nurse should recheck the blood glucose and call the health care provider. The nurse cannot make the decision to
cancel the procedure.
Critical Thinking Activities
Activity 1
49. a. Type 1 diabetes mellitus
b. In addition to polyuria, polydipsia, and polyphagia, she may be thin with a sudden onset of symptoms
including blurred vision, appearance of halos around lights, and headaches. As the condition progresses, there
may be changes in electrolyte balances.
c. Insulin injection are given between the fat and muscle layers.
Gently pinch up at least a 2-inch fold of tissue (not just the skin). And quickly insert the needle into the top
of the fold, entering the subcutaneous tissue. The needle should be inserted at a 90-degree angle. Inject the
insulin slowly. Place the alcohol swab against the needle hub at the injection site, and pull the syringe unit
straight out in one swift motion. Do not massage the site. Teach the patient how to rotate sites for injection.
Store insulin and other supplies properly. Patients can be reminded that aspiration does not need to be done
before injection. The open bottle may be stored at room temperature once opened. It is acceptable to store
unused bottles in the refrigerator.
d. Acute complications include:
• Diabetic coma
• Hyperglycemic hyperosmolar nonketotic coma
• Hypoglycemic reaction
• Increased risk for acute infections
Long-term complications may include blindness, cardiovascular problems, renal failure, and increased risk
of chronic infection (that could lead to amputation). These complications may be avoided or lessened in severity
with the appropriate care and attention to the prescribed medication and dietary regimen.
Activity 2
50. a. Radiographic examinations to determine bone age and a skull series to rule out tumors. Serum growth hormone
levels will also be evaluated.
b. Underdevelopment of the jaw may cause problems with teeth eruption. Sexual development may be delayed.
c. The overall prognosis is favorable. There may be complications with the musculoskeletal and cardiovascular
systems. People with dwarfism can have children with average height.
d. Injection of growth hormone replacement
Activity 3
51. Diabetes mellitus is more prevalent in older adults. A major reason for this is that the process of aging involves
insulin resistance and glucose intolerance, which are believed to be precursors to type 2 diabetes. The classic signs
and symptoms of diabetes may not be obvious in older adults. Older adult diabetic patients are at increased risk for
infection and should be counseled to receive proper immunizations and seek regular medical attention for even
minor symptoms. The older adult often has difficulty managing diabetes. Dietary management may be complicated
by a variety of functional, social, economic, and financial factors.
Some symptoms of hypothyroidism in the older adult are similar to those in a younger person but are more likely
to be overlooked because the symptoms—fatigue, mental impairment, sluggishness, and constipation—are often
attributed solely to aging. The older person with hypothyroidism has more disturbances of the central nervous system,
such as syncope, convulsions, dementia, and coma. There is often pitting edema and deafness.
The older patient with hyperthyroidism frequently has manifestations related only to the cardiovascular system,
such as palpitations, angina, atrial fibrillation, and breathlessness. Signs and symptoms often attributed to “aging”
may actually indicate an endocrine problem.
Activity 4
52. Endocrine disorders can mimic other disorders. For example, palpitations can occur in hyperthyroidism, but can
also occur in cardiac disorders. Older patients especially can have endocrine disorders that cause disorientation,
confusion, or lethargy. These symptoms can be mistaken for other conditions, such as dementia, delirium, drug side
effects, or electrolyte imbalances. Patients may not be able to answer questions about history or symptoms because
of confusion or coma. Symptom development can be subtle or vague and patients themselves may not be aware that
changes are occurring. In addition, many health care professionals are less familiar with endocrine disorders, so
cardiac, respiratory, renal, or nervous system disorders may be suspected before endocrine disorders are
considered.
CHAPTER 12—CARE OF THE PATIENT WITH A REPRODUCTIVE DISORDER
Figure Labeling
1.
See Figure 12-3.
Matching
2.
g
3.
a
4.
e
5.
h
6.
b
7.
i
8.
c
9.
j
10. f
11. d
12. m
13. l
14. n
15. k
Short Answer
16. (a) Producing and storing sperm, (b) depositing sperm for fertilization, and (c) developing the male secondary sex
characteristics
17. See Box 12-2.
18. (a) Amenorrhea: absence of menstrual flow
(b) Dysmenorrhea: painful menstruation
(c) Dysfunctional uterine bleeding (DUB), abnormal uterine bleeding
(d) Menorrhagia: excessive bleeding in amount and duration
(e) Metrorrhagia: bleeding between menstrual periods
19. (a) Unprotected sex, (b) antibiotic resistance, (c) treatment delay, and (d) sexual behavior patterns and
permissiveness
Figure Labeling
20. See Figure 12-12 A.
TABLE ACTIVITY
21. See Table 12-3. Assessment of Risk Factors for Sexually Transmitted Infections, Using the 5 Ps
Multiple Choice
22. Answer 4: If a girl has not started menstruation by age 15 years, or if it has been longer than 3 years since breast
development, a health care provider should be consulted. For onset of menarche, age ranges from as young as 8 to
as old as 15 years. The average age of onset is 12 years.
23. Answer 2: The 3- year old boy is taking social cues from his father about male roles and behavior. Pursuing a same
sex relationship is a manifestation of sexual orientation. The man who is wearing his wife's clothes is a crossdresser or transvestite (sexual orientation is not defined by crossdressing). Investigation of sex change indicates
that the person feels that their sexual identity does not match the biologic body.
24. Answer 3: American Cancer Society recommends an annual mammogram and clinical breast examination for
women (with average risk) starting between ages 40 and 44 years. Obesity is a risk factor for breast cancer and
women over 55 may elect to have annual or every other year examination.
25. Answer 1, 2, 4, 5: Many illnesses—such as diabetes mellitus, end-stage renal disease, hypertension, cancer, certain
types of prostate surgery, spinal cord injuries, organ transplants, chronic obstructive pulmonary disease, and heart
disease or heart surgery—may cause patients concern or may result in actual inabilities with sexual function. In
primary syphilis, there may be a rash or painless chancre, but sexual function is not impaired; thus, the risk to infect
others continues.
26. Answer 1: The American Cancer Society recommends that every woman begin annual Pap tests within 3 years after
becoming sexually active or no later than 21 years of age. Women age 30 years or older may choose to have Pap
screening every 3 years or if combined with HPV screening every 5 years instead of annually. Women who have had
a hysterectomy may stop having cervical cancer screenings (unless their surgery was done as a treatment for cervical
cancer or precancerous cells).
27. Answer 4: Identifying or ruling out pregnancy is the first step. A pelvic examination is likely to be performed,
whether the pregnancy test is positive or negative. The other tests/procedures may be ordered as necessary to
determine cause of amenorrhea.
28. Answer 3: In testicular biopsy, a sample is obtained by aspiration or through an incision into the testes. For semen
analysis, the semen can be obtained by manual stimulation, or by using a condom. The prostatic smear is obtained
by massaging the prostate via the rectum. The prostate-specific antigen is a blood test.
29. Answer 3: Pink-tinged urination, urinary frequency, and burning with urination are considered normal because of
the mechanical irritation caused the scope. The other findings are not expected and could signal infection or other
complications.
30. Answer 2: The pain of “menstrual cramps” that are characteristic of dysmenorrhea can be relieved with local heat
applications or warm showers. In the other conditions, abdominal pain is not anticipated; in addition for excessive
bleeding or irregular bleeding, heat applications could worsen the bleeding.
31. Answer 1, 2, 3, 4, 5: The nurse is assessing for menorrhagia or abnormally excessive bleeding. Comparing flow and
pad/tampon use to regular periods is one way to determine amount of blood loss. Aspirin and anticoagulants could
potentiate blood loss. Rigorous exercise is more likely to be associated with amenorrhea.
32. Answer 1: Patients with premenstrual syndrome (PMS) are taught to keep a symptom diary. These patients can
have anxiety, mood swings, irritability, lethargy, fatigue, sleep disturbances, depression, difficulty concentrating or
forgetfulness. Physical symptoms include: headache, vertigo, backache, breast tenderness, abdominal distention,
acne, paresthesia (burning or tingling) of hands and feet, and allergies. Option 2 includes some of the
signs/symptoms of menopause. Option 3 is a brief description of dysmenorrhea. Option 4 includes some of the
feelings that women who have infertility problems might experience.
33. Answer 2: Premenstrual dysphoric disorder is a severe mood disorder that may be treated with antidepressants.
34. Answer 3: This patient should be referred to the provider, because the bleeding could be a signal of cancer.
35. Answer 2: The hormonal changes that accompany menopause lead to decreased bone density. Calcium and vitamin
D should be encouraged throughout life to support bone health. (See Chapter 4, Medical Management of
Osteoporosis for additional information.)
36. Answer 3: Dyspareunia is pain with sexual intercourse. For postmenopausal women, this could be related to
dryness in the vaginal vault. Pruritus is itching. Dysmenorrhea is painful menstruation. Procidentia is another term
for uterine prolapse.
37. Answer 1: If the patient doesn’t experience any pain, it means that the tubes are occluded, so the gas is not passing
38.
39.
40.
41.
42.
43.
44.
45.
46.
47.
48.
49.
50.
51.
52.
53.
54.
55.
through.
Answer 3: First the nurse tries to help the patient identify what things, events, or factors are making him experience
this sense of losing power. After initial assessment, the nurse may decide to discuss with the patient feelings about
aging, review past accomplishments, or talk about coping strategies.
Answer 1: Sildenafil citrate (Viagra) can potentiate the hypotensive effects of nitrates (nitroglycerin tablets). The
nurse would alert the health care provider so the patient can be properly advised. Vitamin B 6 supplement and
ibuprofen (Motrin) could be prescribed for dysmenorrhea. Cefoxitin (Mefoxin) and corticosteroids are prescribed to
treat pelvic inflammatory disease (PID). Danazol (Danocrine) and vitamin E supplement could be prescribed to treat
fibrocystic breast disease.
Answer 3: For patients with PID, the Fowler’s position facilitates the flow of vaginal drainage.
Answer 2: Flulike symptoms often occur in the first 24 hours. The other symptoms will occur later.
Answer 4: Tampons and pads should be alternated. The use of super-absorbent tampons is not recommended.
Tampons should be changed every 4 hours. The hands should be washed after insertion but washing hands before
insertion is the key to preventing toxic shock syndrome.
Answer 1: Vesicovaginal fistula causes a constant trickling of urine into the vagina; a rectovaginal fistula allows
feces and flatus to enter the vagina. Uterine prolapse creates a subjective sensation of "something falling down".
Answer 3: Radiation therapy is usually started 2-3 weeks after surgery, when the wound is completely healed and
the patient can comfortably raise her arm over her head.
Answer 2: The technique uses a balloon catheter to insert radioactive seeds into the breast after the tumor is
removed (at the time of the lumpectomy or shortly thereafter into the tumor resection cavity). In brachytherapy, an
internal radiation therapy, the patient is hospitalized for 48 hours. For external radiation, the treatments are usually
done 5 days a week for 5-6 weeks
Answer 1: Epoetin alfa (Procrit) is helpful in raising erythrocyte counts to help correct anemia. The other drugs
could be ordered to control the nausea and vomiting associated with chemotherapy.
Answer 1, 3, 4: Tamoxifen is not used for women who desire continued fertility. For postmenopausal women, it
increases the risk of blood clots, cataracts, and endometrial cancer. Toremifene (Fareston), is indicated as first-line
treatment for metastatic breast cancer in postmenopausal women with estrogen receptor–positive or estrogen
receptor–unknown tumors. The other statements apply to tamoxifen.
Answer 2: Autologous indicates originating within self; thus, the patient donates the bone marrow; then high doses
of chemotherapy are given to suppress the bone marrow. The patient subsequently undergoes autologous bone
marrow or stem cell transplantation. Radiation and plasmapheresis are not used.
Answer 2: A cone-shaped section will be cut from the cervix; thus, it is important to monitor for bleeding after the
procedure. Schiller’s iodine test is used for the early detection of cancer cells and to guide the health care provider in
doing a biopsy. Encouraging fluids is done prior to ultrasound. Refraining from powders, deodorants, or ointments
is an instruction given for mammography.
Answer 3: Oral contraceptives may be used to suppress ovulation by inhibiting prostaglandin levels. A recent theory
proposes that dysmenorrhea may be caused by hypercontractility of the uterus resulting from higher-than-normal
levels of prostaglandins.
Answer 4: The first sign of endometrial cancer is abnormal uterine bleeding, usually in postmenopausal women.
About 50% of patients with postmenopausal bleeding have cancer of the uterus.
Answer 2: To distinguish a hydrocele from a cancerous testicular mass, a strong light is directed from a point behind
the scrotum (transillumination), the light passes through if the swelling is a hydrocele; if the swelling is a solid mass
the light does not pass through.
Answer 1: Parenteral benzylpenicillin (penicillin G) remains the treatment of choice for all stages of syphilis. In
patients who have an allergy to penicillin, tetracycline, erythromycin and ceftriaxone are prescribed.
Answer 4: In the male, signs and symptoms of gonorrhea are mild to severe transient urethritis, dysuria, frequent
urination, pruritus, and purulent exudate. Genital herpes is characterized by recurrent episodes of acute, painful,
erythematous, vesicular eruptions (blisters) on or in the genitalia or rectum. The first sign of primary syphilis is a
painless erosion or papule that ulcerates superficially with a scooped-out appearance. In men, signs and symptoms
of chlamydia may include a scanty white or clear exudate, burning or pruritus around the urethral meatus, urinary
frequency, and mild dysuria.
Answer 2: Patients should not drink alcoholic beverages, since alcohol and metronidazole (Flagyl) can interact and
56.
57.
cause reactions such as disorientation, headache, cramps, vomiting, and possibly convulsions.
Answer 3: The girl is showing signs/symptoms of toxic shock syndrome. The nurse knows that risk for toxic shock
increases if tampons are left in place too long, or if they are inserted with fingers or under unhygienic conditions.
The first action is to remove the tampon and then immediately seek medical attention.
Answer 3: Pessaries are placed for uterine support. They should be removed and cleaned every 3-4 months.
Unattended pessaries can cause erosion, fistula, and carcinoma.
Critical Thinking Activities
Activity 1
58. a. Genital herpes
b. There is no cure for herpes. Antiviral medications [acyclovir (Zovirax), valacyclovir (Valtrex) and famciclovir
(Famvir)] can be prescribed to manage the initial outbreak which may last from 3-10 days. Suppressive therapy
includes continued use of oral acyclovir for up to 5 years, but it should be interrupted after 1 year to assess the
recurrence rate. Local anesthetics (e.g., lidocaine) or systemic analgesics (e.g., codeine and aspirin) may be
administered.
c. Advise the patient to keep genital lesions clean and dry. Hands should be washed thoroughly after touching a
lesion. Loose, absorbent underclothing is usually more comfortable than close-fitting clothing. Sitz baths
decrease discomfort and enhance urinary and bowel elimination. Sexual intercourse during the active lesion
phase increases the risk of transmission and may also be painful. Patients should inform future sexual partners
(use barrier methods, especially condoms) and health care providers of recurring or latent infections. In the
event of pregnancy, the disease must be monitored closely; there is a possibility of spontaneous abortion.
Activity 2
59. a. Menarche begins on average at age 12, but the age range is 8-15.
b. On days 1 through 5 of the cycle (the menstrual phase), the endometrium sloughs off, accompanied by 1 to 2
ounces (30-60 mL) of blood loss.
c. Personal hygiene
• Wear pads during heavy flow.
• Change tampons every 4 hours to decrease risk of toxic shock syndrome.
• Consult health care provider if tampon use frequently causes discomfort.
• Take a daily shower for comfort; warm baths may relieve slight pelvic discomfort.
• Keep perineal area clean and dry; cleanse from anterior to posterior.
• Wear cotton underwear; remember that tight nylon underwear and tight-fitting jeans retain moisture and
should not be worn for extended periods.
• Feminine hygiene products such as vaginal sprays and suppositories may contribute to a feeling of
cleanliness.
• A daily douche is not recommended because it changes the protective bacterial flora of the vagina and
predisposes the woman to infection.
Activity 3
60. a. Risk factors for sexually transmitted infections include: younger age, single, urban, poor, male, or homosexual,
frequent sexual contact with multiple partners, and unprotected sexual activity. Poor hygiene and poor nutrition
are more likely to occur for the homeless and both contribute to infection. Poor nutrition also contributes to
problems with menstruation.
b. Until personal values are challenged, it is difficult to know exactly how one will react or cope. Having as much
information about the new job, the patient population, and self is one way to prepare. Having support systems
in place (family, friends, colleagues) is another way to prepare for new experiences.
With regard to gender identity differences, the nurse is likely to encounter gender issues in a large city that
she never saw in her small hometown. In the beginning, the nurse may wonder, “Should I use Mr. or Ms. when I
am addressing this androgynous person?” “Should I direct this person to the women’s restroom or the men’s
restroom?” The nurse will learn to deal with these kinds of questions by relying on the principles of therapeutic
communication. “How would you like me to address you?” “The restrooms are over there to the right and the
left.”
One of the more difficult aspects of being a nurse is trying to be nonjudgmental towards patients who
contribute to their own health problems by repeatedly participating in risky behaviors; thus, if the nurse sees the
same young woman repeatedly return to the clinic to be treated for STIs, the nurse may think, “What’s the use?”
In order to continue in this job, the nurse will have to examine her own beliefs and value system to determine if
she can sustain commitment to the patient’s right of self-determination and continue to offer accurate
information and compassionate care.
Activity 4
61. The decision to have a child is possibly the most important decision that people make and inability to conceive
creates self-doubt. Diagnostic testing can produce a great deal of anxiety and stress. This testing may continue for
fairly long periods with or without favorable results. Infertility testing can be expensive and may not be covered by
some insurance carriers. Feelings of anger, frustration, sadness, and helplessness between partners and between the
couple and health care providers may increase as more tests are performed.
There are many factors that can possibly contribute to infertility. Some of these relate to lifestyle, such as
smoking, excessive alcohol use, athletic training, obesity, being underweight, or deciding to delay childbearing.
These factors can produce guilt and contribute to anxiety.
CHAPTER 13—CARE OF THE PATIENT WITH SENSORY DISORDERS
1.
See Figure 13-1.
Short Answer
2.
a. Refraction: light rays are bent as they pass through the colorless structures of the eye, enabling light from the
environment to focus on the retina.
b. Accommodation: the eye is able to focus on objects at various distances. It focuses the image of an object on the
retina by changing the curvature of the lens.
c. Constriction: the size of the pupil, which is controlled by the dilator and constrictor muscles of the iris, regulates
the amount of light entering the eye.
d. Convergence: medial movement of both eyes allows light rays from an object to hit the same point on both
retinas.
3.
a. Total blindness is defined as no light perception and no usable vision.
b. Functional blindness is present when the patient has some light perception but no usable vision. It may be
congenital or acquired.
c. Legal blindness refers to individuals with a maximum visual acuity of 20/200 with corrective eyewear and/or
visual field sight capacity reduced to 20 degrees.
4.
a. In conductive hearing loss, sound is inadequately conducted through the external or middle ear to the
sensorineural apparatus of the inner ear.
b. In sensorineural hearing loss, sound is conducted through the external and middle ear in a normal way, but a
defect in the inner ear results in distortion, making discrimination difficult.
c. Mixed hearing loss is a combined conductive and sensorineural hearing loss.
d. Congenital hearing loss is present from birth or early infancy.
e. Functional hearing loss may be caused by an emotional or a psychological factor.
f. Central hearing loss occurs when the brain’s auditory pathways are damaged, as in a stroke or a tumor.
5.
a. Sweet: Respond to sugar and other sweet substances; located on the tip of the tongue
b.Sour: Respond to acid content of foods; located on the sides of the tongue
c. Salty: Respond to metal ions within foods; located on the tip of the tongue
d. Bitter: Respond to alkaline or basic ions within foods; located on the posterior portion of the tongue
Figure Labeling
6.
See Figure 13-13.
Multiple Choice
7. Answer 3: According to the American Foundation for the Blind, a functional vision assessment investigates visual
functionality for near task (closer than 16 inches), intermediate task (16 inches to 3 feet) and then a distance task
(more than 3 feet). If a patient failed the distance task, driving would not be safe.
8.
Answer 2: The automated perimetry test is a test for peripheral vision. Participating in a rapid action sport, such as
basketball, would be difficult if vision in the outer fields is limited. The other tasks require a more focused view of
what is straight ahead.
9.
Answer 3: During fluorescein angiography, a dye is injected into a vein. The dye could cause a similar allergic
reaction for those who react to seafood or iodine.
10. Answer 4: Corticosteroids are contraindicated in keratitis (inflammation of the cornea) because they contribute to a
longer course, possible deeper ulceration of the cornea, and systemic complications. Corticosteroids may be
prescribed for the other disorders.
11. Answer 2: Diplopia is double vision, so reading is going to be very difficult, if not impossible. The patient should be
instructed to steady self by grasping the bed rail or the arm of the chair when sitting upright. Foods that can be
eaten with the fingers will be easier for this patient. Listening to the radio would be a better distraction than
watching television.
12. Answer 2, 4, 5: The purpose of the cane is to determine the boundaries of the walking path and the tip of the cane is
used to seek anything obstructing the path. The helper should walk in front of the patient; patient can hold the
elbow for security and to detect directionality of helper’s movements. Walking slowly is advised so that objects can
be detected. Descriptions of surroundings help to create a mental picture for the patient.
13. Answer 4: In hyperopia, the patient can see distant objects, but close objects such as fine print are blurry; using overthe-counter eyewear that magnifies fine print may work initially.
14. Answer 2: Contact lenses change the shape of the cornea, so for a week or two prior to the initial evaluation, the
health care provider will ask the patient not to wear them. Usually one day is sufficient for rest after surgery.
Possibly, anticoagulant medications would be held, but systemic complications related to refractory surgery are
unlikely.
15. Answer 3: People who wear contact lenses know they are not supposed to use saliva to clean the lenses; however,
many users forget to carry sterile solution or a spare contact case. The nurse should help contact lens users plan
ahead. Borrowing solution or lens cases from others is not recommended because of risk for infection. Adolescents
generally prefer not to wear glasses, but possibly for active sports they are preferable.
16. Answer 3: Use of fresh makeup, individual applicators, and supervising the activity is the best option. This may
seem a little costly, but the alternative would be to ban the activity with an explanation about eye infections.
17. Answer 4: Eye pads are contraindicated because they facilitate bacterial growth. The other actions are correct.
18. Answer 2: Severe eye pain is associated with this disorder.
19. Answer 1: Sjögren syndrome is an immunologic disorder characterized by deficient fluid production by the lacrimal,
salivary, and other glands, resulting in abnormal dryness of the mouth, eyes, and other mucous membranes.
20. Answer 4: The eyes feel gritty because of the deficient fluid production in glands of the mouth, eyes, and other
mucous membranes.
21. Answer 1, 2, 4, 5: Ectropion and entropion are characterized by abnormal direction of the eyelid with tearing and
corneal dryness. Redness of the sclera may also be present.
22. Answer 3: The health care provider will use visual inspection and an ophthalmoscopic examination. Amsler’s grid
assesses for disturbances in central vision. Snellen’s test assesses visual acuity. Pneumatic retinopexy is a procedure
used to correct retinal detachment.
23. Answer 2: In diabetic retinopathy, microhemorrhages will cause floaters.
24. Answer 1: This patient is reporting symptoms of macular degeneration.
25. Answer 3: Tonometry uses puffs of air forced into the open eye. An increased ocular pressure suggests glaucoma.
26. Answer 1: Photophobia, dryness, burning, or tearing should be reported to the health care provider. The other
statements are correct.
27. Answer 2: Lifting, bending, coughing, or stooping would increase intraocular pressure, which is not desirable in the
postoperative period. The surgery should improve the glare that would occur while watching a movie. Sunglasses
are recommended. Sexual activity may be unadvisable for a period of time. Sleeping with a spouse would be okay
unless he/she tended to thrash around during sleep (risk for trauma to eye).
28. Answer 1: High-dose nutritional supplements of zinc, beta-carotene, and vitamins C and E have been shown to
29.
30.
31.
32.
33.
34.
35.
36.
37.
38.
39.
40.
41.
42.
reduce the risk of progression in age-related macular degeneration. A diet rich in fruits and dark-green leafy
vegetables is also recommended.
Answer 2: Progressive enlargement of the darkened area means the detachment is worsening and if the retina is not
repaired, irreversible blindness will result. Pain is not an expected symptom of detachment. Type 1 diabetics are at
risk for diabetic retinopathy and there is an increased risk for cataracts. Retinal detachment can be related to injury,
but is mostly related to aging, not heredity.
Answer 1, 3, 4, 5: The current recommendation is for an ophthalmologic examination every 2 to 4 years for people
between 40 and 64 years of age, and every 1 to 2 years for people 65 years of age or older. African Americans in
every age group should have more frequent examinations because of the increased incidence of glaucoma. Nearly
all patients with type 1 diabetes and 80% with type 2 diabetes have some degree of retinal disease; therefore,
frequent examinations are recommended. People who wear contact lenses are generally advised to have eye
examinations every 1 to 2 years.
Answer 3: Cotton is not used because of potential to scratch the cornea. The other methods are acceptable.
Answer 2: The eye and stick are covered with a cup to prevent dislodgment (cup should be sufficiently large to
cover the stick without touching it). Uninjured eye is patched to reduce consensual movement. Then the camper is
taken to the hospital if 911 is not available to respond to the camping site.
Answer 3; 0rgan of Corti, known as the organ of hearing contains many fine hair cell receptors, which respond to
sound waves. Once these cells are damaged or destroyed, hearing is impaired.
Answer 4: The student is trying to help the patient, but the nurse must clarify that not all types of hearing loss are
corrected by a hearing aid. The nurse could use the other options to help the patient and student explore topics
related to hearing aids and communication.
Answer 2: In conductive hearing loss, sensitivity to sound is diminished, but clarity or interpretation of sound is not
changed. Increased volume or amplification compensates for the conductive loss; therefore, a hearing aid may be
helpful.
Answer 4: If the Romberg test is abnormal, the patient lost his balance when standing erect, feet together, with eyes
closed.
Answer 2: A warm compress over the affected ear will help relieve the pain. Swallowing can relieve the pressure but
sobbing and swallowing increase the chance for vomiting. The acetaminophen will work but recall that pain
medication is not as effective if given during the peak of pain. A prescription for a sedative is possible if the pain
and sleeplessness are excessive.
Answer 1: Ear pain and pulling at the ear are early signs/symptoms of acute otitis media.
Answer 2: With tinnitus the patient hears ringing or buzzing. Back ground music may provide relief. A dark,
quiet room and lying very still help to relieve symptoms of vertigo. Television or flickering lights can worsen
symptoms of Meniere’s disease.
Answer 2: Meclizine (Antivert) is a medication used in the treatment of vertigo, which causes dizziness and a
sensation of spinning.
Answer 1: Keep the patient flat with the operative side facing upward to maintain the position of the prosthesis and
graft; make certain that the patient is not turned.
Answer 1: Proprioception (sense of position) maintains the proper position of the body. Proprioceptors work in
conjunction with the semicircular canals and the vestibule of the inner ear to maintain proper coordination, allowing
body's movements, such as running, walking, or dancing.
Critical Thinking Activities
Activity 1
43. a. Monitor pressure dressing over eye. The dressing should be inspected at least every hour.
Assess for pain on the affected side or any headache. Monitor vital signs.
b. Excessive bleeding from site, headache, signs/symptoms of excessive blood loss
c. Encourage verbalization of specific concerns. Provide support. When appropriate, advise patient that with
healing, he can be fitted with a prosthetic device in 4-6 weeks.
Activity 2
44. a. Mastoiditis
b.
c.
It is the result of a spreading middle ear infection. The patient’s risk was enhanced after not completing the
prescribed antibiotic therapy.
If caught early, treatment will include IV antibiotic therapy and a myringotomy. If the infection has progressed,
treatment will include IV antibiotic treatment and a simple mastoidectomy.
Activity 3
45. Nursing interventions for the patient having a vitrectomy include:
• Helping the patient to maintain a position on the abdomen or sitting forward resting the nonoperative side of
the head on a table to allow air that is in the eye to float against the retina. This position is maintained for 4 to 5
days.
• Assessing the eye patch
• Applying ice packs
• Monitoring vital signs
• Assessing the dressing for bleeding
• Dark glasses are prescribed postoperatively to decrease the discomfort of photophobia.
Activity 4
46. It is likely that you have a grandparent, parent, or older aunt or uncle who has demonstrated some of the behaviors
associated with hearing loss. The symptoms may have been gradual or only a few may have occurred so far. There
may be circumstances where the behaviors are more pronounced. Most people adapt to gradual losses and loss of
hearing may be more noticeable to those around who are trying to communicate with that person.
Activity 5
47. A sudden loss of any of the senses is devastating. Since you are currently in nursing school, the loss would impact
your ability to complete your studies. Moreover, imagine how difficult it would be to assess a patient if you couldn’t
see or hear. Would you be able to perform patient care if you couldn’t see? How would you administer medication if
you couldn’t read the label?
Perhaps you have small children and they rely on you for everything. How would you adapt and cope so that
the impact of your loss did not adversely affect them?
CHAPTER 14—CARE OF THE PATIENT WITH A NEUROLOGIC DISORDER
Figure Labeling
1.
See Figure 14-2.
Matching
2.
e
3.
f
4.
a
5.
c
6.
i
7.
g
8.
b
9.
h
10. j
11. d
12. k
Figure Labeling
13. See Figure 14-7.
Word Scramble
14.
15.
16.
17.
18.
Alert e
Disorientation d
Stupor a
Semicomatose b
Comatose c
Multiple Choice
19. Answer 3: A person with Parkinson's disease has decreased dopamine levels and suffers involuntary, trembling
20.
21.
22.
23.
24.
25.
26.
27.
28.
29.
30.
31.
32.
33.
34.
muscle movements, or tremors. Any shift in the balance of activity (an increase in acetylcholine or a decrease in
dopamine) seems to lead to parkinsonian-like symptoms.
Answer 2: Global aphasia, the inability to understand the spoken word or to speak, would present the greatest
communication challenge. In receptive aphasia, patient could communicate needs, but would not understand a
verbal or written response. In motor aphasia, the patient can't speak or write, but would understand written or
spoken responses. In anomic aphasia, the patient could indicate a need, but not be able to name objects.
Answer 1, 2, 3, 4, 6: Changes related to aging include slowed reaction time, slowed learning, slight tremors when
fatigued, increased difficulty with fine motor movement, and short-term memory loss. Nonpurposeful action like
shuffling items is associated with dementia. Ability to locate misplaced items demonstrates a retention of problemsolving ability, despite some forgetfulness.
Answer 4: Fund of knowledge is an assessment of the patient’s retention of general knowledge that the average
adult should know. The other components are orientation to time, person, and place; assessment of short-term
memory; and ability to calculate.
Answer 4: The patient is demonstrating the maximum possible score which is 15 total points.
Answer 3: The FOUR Score coma scale includes eye response, brainstem reflexes, motor response, and respiration.
Answer 3: In motor aphasia, the patient can understand the nurse, but is unable to use the symbols of speech; thus,
pointing at pictures or objects and developing a language of gestures will help the patient.
Answer 4: The glossopharyngeal and vagus are involved in the gag reflex and swallowing movements. The
trochlear and abducens nerves are involved in eye movement. Trigeminal is involved in jaw strength, facial
sensation, and corneal reflex. Facial is involved in taste and facial movements.
Answer 1: In unilateral neglect, the patient is unaware or inattentive to one side of the body; thus, she is unlikely to
be able to accomplish any task that requires two hands. It is possible that she would struggle to put on one sleeve.
Answer 4: UAP is not expected to assess for numbness or tingling but should be instructed to report any patient
complaints of numbness, tingling, or pain. The patient should be flat in bed and fluids are usually encouraged. Both
measures are to prevent headaches.
Answer 3: If the access is at the carotid artery, a hematoma or swelling could cause an airway obstruction.
Respiratory effort is the priority assessment. Infection is always a concern, but there are no signs immediately after
the procedure. Delayed reaction to contrast medium is possible, but usually the chief concern for contrast media is
immediately after administration. Nausea and vomiting might occur, but usually nausea will occur in response to
the contrast medium and that sensation is generally mild and transient.
Answer 1: The health care provider is likely to suggest acetaminophen, phenacetin, ibuprofen, and aspirin. Opioids
are avoided because these drugs are often subject to abuse; it is much better to counsel patients to develop other
ways to relieve headaches. The nurse should suggest nonpharmaceutical measures such as relaxation techniques,
regular exercise, adequate sleep, and avoidance of alcohol.
Answer 4: Many foods may contribute to migraines: such as aged cheeses (cheddar and Swiss), cured meats,
fermented cabbage (sauerkraut), and soy and fish sauces. Nitrites are present in curing substances used in the
preparation of meats such as bologna, ham, hotdogs, and bacon. Other substances that may provoke headaches
include vinegar, chocolate, yogurt, alcohol, fermented or marinated foods, and caffeine.
Answer 2: The patient is likely to be more comfortable in a quiet, dark room. The patient can turn self. Warm
compresses are not needed. Patient may refuse foods and liquids during the peak of nausea, but does not need to be
kept on nothing by mouth status.
Answer 3: Gabapentin (Neurontin) is a medication that is prescribed for neuropathic pain. Diabetics frequently have
this type of pain in the lower part of the legs.
Answer 1: Change in level of consciousness is an early sign. The others are late signs.
35.
36.
37.
38.
39.
40.
41.
42.
43.
44.
45.
46.
47.
48.
49.
50.
51.
52.
Answer 1: All of the patients have signs/symptoms of possible increased intracranial pressure; however, rise in
systolic pressure and an unchanged diastolic pressure, resulting in a widening pulse pressure, bradycardia, and
abnormal respiration (Cheyne-Stokes respiration), are late signs of increased intracranial pressure and indicate that
the brain is about to herniate.
Answer 1: A sluggish reactive pupil can be difficult to recognize, but it is the first pupillary sign in increased
intracranial pressure. The fixed and dilated pupil is the most ominous sign, which warrants immediate notification
of the health care provider. None of these reactions are considered normal and all should be reported to the
provider.
Answer 3: Patient with diabetic neuropathy has decreased sensation in lower extremities, so visual inspection is
way to compensate for that deficit. Patient with agnosia (inability to recognize familiar objects by sight, touch, or
hearing or to recognize familiar people through sensory stimuli) needs ongoing assistance. Patient with loss of
proprioception (knowing position of body in space) may additional assistance to maintain safety with walker.
Patient who is speaking loudly should be referred to a provider for evaluation of hearing loss.
Answer 3: Patients with paraplegia are usually taught to use the upper body to compensate for loss of movement in
the lower extremities. Weight shifts include controlled leaning from one side to another or chair push-ups.
Answer 2: In counterpositioning to protect the upper extremity, the patient is placed so that the shoulder and upper
arm are in abduction, the elbow is flexed, the wrist is dorsiflexed.
Answer 1: Phenytoin (Dilantin) is used to prevent seizures. Medication compliance is an important to maintain a
therapeutic level.
Answer 1, 3, 5: Fluid is restricted to avoid adding fluid volume to the system. Flexion of the hips increases
intraabdominal and intrathoracic pressure. Oxygen is given to support impaired brain tissue. Head should be in a
neutral position. Enemas are not recommended.
Answer 2: The affected arm should be put through range-of-motion exercises. It is unlikely that the patient can walk
safely to the bathroom, even with assistance. The prone position would be good for the patient, but the nurse should
make the determination if the patient can tolerate it, rather than expecting the UAP to make that decision. In
hemiplegia, the upper arm will tend to fall forward, so the counter-position is abduction.
Answer 1, 2, 3, 4: Multiple sclerosis is a disease that more frequently develops in young women. The onset is
insidious, the symptoms are vague, and there are bouts of exacerbation and remission, but with progressive
deterioration. The patient will be discouraged, because many treatments will have been tried, some will give partial
symptom relief, but there is no cure and the patient sees herself getting progressively worse to the point of being
totally helpless.
Answer 1: The classic triad of Parkinson’s includes tremors, rigidity, and bradykinesia. Bradykinesia affects the gait
and he may be propelled forward until an obstacle stops him. Stiffness in bending or moving the arms is a sign of
rigidity. Tremors affect fine motor control.
Answer 1: Withdrawal of the Parkinson's medications could herald the return of all of the symptoms, but the
priority concern is aspiration.
Answer 2: Eyelid drooping and double vision are considered early signs. The other signs will come later as the
disease progresses.
Answer 4: Respiratory problems are typical; frequent upper respiratory tract infections occur because the patient
may not have the energy to cough effectively, and pneumonia or airway obstruction may develop. Aspiration often
occurs. A severe exacerbation can result in death from respiratory failure.
Answer 3: Stroke risk can be reduced by up to 42% with appropriate treatment of hypertension. Controlling the
other factors will also reduce risk.
Answer 4: The nurse would check for unintentional pouching of food on the affected side of the mouth. The other
options are incorrect, except use of covered cups is acceptable.
Answer 1: For thrombolytic therapy, the timing is critical to the outcome. The clinic staff should work towards
immediate transfer to a stroke center. If the patient were to suddenly become unresponsive, the clinic staff would
stop to intervene; otherwise no action should delay transfer to a stroke center.
Answer 3: The patient may prefer to do his own care, because the face is very painful, and he may fear that the UAP
will cause pain just by touching. Shaving, combing hair, and hygiene in general can be deferred until the pain is
better controlled. Warm puréed foods are best. Cold liquids are likely to increase pain.
Answer 2: Bell’s palsy is an inflammation of the facial nerve and the muscles of the face of the affected side become
53.
54.
55.
56.
57.
58.
flaccid. This includes the eyelid. The purpose of the eye shield at night is to prevent corneal damage because the
eyelid will not stay closed.
Answer 2: The weakness and paralysis will start in the legs and move upwards. The primary concern is that rapid
progression upwards will cause paralysis of the respiratory muscles.
Answer 3: For this patient, the reduction of stimuli decreases the risk for seizures, which are a complication of
meningitis. The other options are correct rationales for different patient conditions.
Answer 2: Headaches are the most prominent early sign. Patients often report that the headache is more severe in
the morning.
Answer 1: Redirection is the best first action, because it is possible that the nurse can get him to focus on something
else. Medicating him is possible, but is not the first action to try, because it would be considered a chemical restraint.
Allowing him to wander is a possibility, but his agitation could increase. Assigning UAP to do one-to-one
observation is also possible if the nurse believes that the resident is a danger to himself (nurse would call provider,
as soon as possible, to obtain an order for one-to-one observation).
Answer 4: Putting the patient in a sitting position decreases the blood pressure, especially the pressure in the head.
Bladder distention and fecal impaction are the most common causes, so the nurse would check these and try to
resolve the issue. The nurse can direct the UAP to recheck the blood pressure. This is a medical emergency and if the
pressure does not come down, the health care provider must be notified so that drug therapy can be started.
Answer 4: See Table 14-8 for additional information.
Critical Thinking Activities
Activity 1
59. a. The nurse stays with the child and the area is cleared of dangerous objects if possible. The child’s head is
supported and protected and if possible, turned to the side to maintain the airway. Restrictive clothing around
the neck is loosened. The child is not restrained and no objects are placed in the mouth.
b. The nurse would note, record, and report events that preceded the seizure, presence of aura, when the seizure
occurred, length of ictal phase and postictal phase, and what occurred during each phase.
Activity 2
60. a. Transient ischemic attack (TIA)
b. Yes, TIAs are significant because at least one in three people who experience TIAs will experience a
cerebrovascular accident within 2-5 years.
c. Aspirin
Activity 3
61. a. See Box 14-2 for the Warning Signs of Alzheimer’s Disease (AD).
b. Currently no effective treatment is available to stop the progression of AD, which occurs at a variable rate. The
course of the disease can span 5-20 years. The economic costs of AD in the United States is on average $ 61,522
annually. While portions of this cost are absorbed by insurance coverage, large costs are borne by the family.
Ultimately, most patients die from complications such as pneumonia, malnutrition, and dehydration. The
burden on the individual, the family, caregivers, and society as a whole is staggering.
c. Engage in activities that require information processing (e.g., reading, learning a new language, doing
crossword puzzles). Participate in regular physical activity, leisure activities, and educational achievements
throughout the lifespan. Antioxidant-containing foods such citrus fruits, dark-green vegetables, tomatoes,
brown rice, and foods high in beta-carotene (sweet potatoes and carrots) are considered to lower the risk of the
development of AD.
CHAPTER 15—CARE OF THE PATIENT WITH AN IMMUNE DISORDER
Figure Labeling
1.
See Figure 15- 2.
Short Answer
2.
3.
4.
5.
(a) To protect the body’s internal environment by destroying foreign antigens and pathogens, (b) to maintain
homeostasis by removing damaged cells from the circulation, and (c) to serve as a surveillance network for
recognizing and guarding against the development and growth of abnormal cells.
a. Recognize self from nonself
b. Respond to nonself invaders
c. Remember the invader
d. Regulate its action
See Box 15-2 for additional information.
a. Host response to allergen: The more sensitive the individual, the greater the allergic response is.
b. Exposure amount: Generally, the more allergen the individual is exposed to, the greater the chance of severe
reaction.
c. Nature of the allergen: Most allergic reactions are precipitated by complex, high-molecular–weight protein
substances.
d. Route of allergen entry: Most allergens enter the body via gastrointestinal and respiratory routes. Injections of
venoms and medications hold a more severe threat of allergic response.
e. Repeated exposure: Generally, the more often the individual is exposed, the greater the response is.
In addition to gloves, latex-containing products used in health care may include blood pressure cuffs, stethoscopes,
tourniquets, IV tubing, syringes, electrode pads, oxygen masks, tracheal tubes, colostomy and ileostomy pouches,
urinary catheters, anesthetic masks, and adhesive tape.
Multiple Choice
6. Answer 4: The innate system (first line of defense) includes intact skin and mucous membranes, cilia, stomach acid,
tears, saliva, sebaceous glands, and secretions and flora of the intestine and vagina. Breast feeding gives the infant
passive immunity. Immunization provides active artificial immunity. Antibiotics are used to treat bacterial
infections, but do not provide any type of immunity.
7.
Answer 2: Surgical removal of lymph nodes, thymus, or spleen can suppress the immune response. Splenectomy in
children is especially dangerous and may lead to septicemia from simple respiratory tract infections .
8.
Answer 3: The nurse recognizes that the elderly residents have a greater risk for infection, especially pneumonia
and urinary tract infection. The nurse would check the respiratory system first (based on Maslow's hierarchy and
the ABCs [airway, breathing, circulation]). The nurse would assess the urine and ask about urinary patterns. For
complete vital sign data, the nurse would check the temperature, but elderly people may or may not have a
temperature elevation with infection. Reporting findings to the provider and obtaining orders for diagnostic testing
are also appropriate.
9. Answer 1: Human Papilloma Vaccination (HPV) is recommended for women 19-26 years of age and for men 19-21
years of age –3 doses if not previously vaccinated. If having sex with men, vaccine should be given for men 19-26 if
not previously vaccinated.
10. Answer 4: The nurse would instruct the UAP to wash and dry hands after removing latex gloves. Avoiding latex
exposure is difficult in the current health care environment. The staff needs education and training to identify latex
sources and signs/symptoms of type I (occurs within minutes) and type IV (occurs with 6-48 hours) allergic
reactions. The nurse should assess patients for latex allergies, inform UAP, obtain a latex-free supply cart (if
available), and post an allergy alert sign. Use of gloves is not based on the visibility of blood or body fluids, but on
the possibility of exposure.
11. Answer 2: If the patient keeps a food diary, this data can be used to identify possible allergens. Once allergens are
identified, the health care provider can prescribe therapy.
12. Answer 1, 2, 3, 5, 6: Older adults are prone to urinary tract infections and urinary stasis will contribute. Fluids are
offered to thin secretions because older adults have trouble coughing up secretions. Skin becomes fragile and dry.
Hand hygiene is always appropriate; older adults have increased risk for infection. Oral hygiene is important
because saliva (which fights bacteria) is decreased. Teaching and assessing "as needed" are nursing responsibilities.
The UAP could encourage coughing and deep breathing after the nurse gives appropriate instructions.
13. Answer 2: Progressively increasing the dose of allergens over time allows the individual to build up a tolerance, but
not have the symptoms, because the initial dose is very dilute. Leukotriene inhibitors such as montelukast
(Singulair) are agents that significantly reduce symptoms of an allergic reaction caused by the release of leukotrienes
14.
15.
16.
17.
18.
19.
20.
21.
22.
23.
from mast cells and basophils. Antihistamines compete with histamine by attaching to the cell surface receptors and
blocking histamine release. Epinephrine produces bronchodilation and vasoconstriction and inhibits further
release of chemical mediators of hypersensitivity reactions from mast cells.
Answer 3: Intravenous administration of medication is most likely to produce a rapid reaction if the patient has
allergies to the medication, because the circulatory system will rapidly distribute the drug throughout the body. In
the other routes, the absorption will be delayed compared to the IV route.
Answer 2: With friends, the nurse may be tempted to joke, but apparently this individual does not understand the
physiology of allergic response. Every exposure to oysters has the potential to create a more rapid and rigorous
response. Taking diphenhydramine (Benadryl) may seem like a preventive measure to the friend, but abstinence is a
better solution.
Answer 4: The nurse could try any of these strategies, but the patient is not able to clearly communicate or report on
the complex factors in the home setting. The home health nurse can assess the situation and make recommendations
that are specific to the patient's environment. (Note to student: Use critical thinking to determine the best
interventions for patients; in this case, making a referral.)
Answer 1: A urine specimen is obtained to assess for hemolysis.
Answer 4: Immunoglobulin levels decrease with age and therefore lead to a suppressed humoral immune response in
older adults.
Answer 1, 2, 3, 5: The plasma is generally replaced with normal saline, lactated Ringer’s solution, fresh frozen
plasma, plasma protein fractions, or albumin.
Answer 2: Immediate aggressive treatment is the goal in anaphylaxis. At the first sign, 0.2 -0.5 mL of epinephrine
1:1000 is given subcutaneously for mild symptoms. The other actions may also be needed if the symptoms
progress.
Answer 1, 2, 3: Breastfeeding provides natural passive immunity for the baby. Antivenom after a snakebite and
postexposure immunoglobulin provide artificial passive immunity. Having a disease like measles provides natural
active immunity and getting vaccinated provides artificial active immunity.
Answer 2: The patient can have visitors, but ideally the nurse should screen all visitors for potential minor
infections, remind them about handwashing, and check to make sure that no potentially infectious items or gifts are
brought to the patient. Seven to 10 days is the time for tissue rejection; the UAP is not responsible for knowing how
to respond or check for this. An instruction, such as to report pain, could be given. The patient’s medications should
not harm a pregnant UAP or the fetus. Health care staff with a cough or skin infection should not enter the room,
even with mask and gown, if there are alternative team members who could be assigned.
Answer 2: Hypotension and citrate toxicity, which may cause hypocalcemia (headache, paresthesias and dizziness),
are the most common complications.
Critical Thinking Activities
Activity 1
24. a. The patient should be monitored after the allergy shot. This monitoring should include observation for adverse
reactions, for at least 20 minutes.
b. The patient should be taught signs and symptoms to look for regarding hypersensitivity reactions. The patient
should always have an EpiPen and know how to use it and when to call 911 or the health care provider.
c. The health care provider should be notified. Interrupted doses put the patient at risk for hypersensitive
reactions.
Activity 2
25. a. As a normal part of aging, a person’s immune system will often weaken. The risk of inflammation and infection
increases with age. Skin becomes more fragile and may allow pathogens to enter. Infection in most body
systems also increases due to decreased production and movement of secretions. Aging often brings on diseases
and disorders of several body systems. These may further complicate the patient’s health status.
b. The importance of handwashing, avoiding potentially harmful situations (e.g., crowded conditions), and the
need for influenza, pneumonia and zoster immunizations should be addressed. Based on assessment findings,
the nurse may decide to review the general principles of maintaining good health: balanced nutrition, adequate
sleep and rest, and patient/age appropriate exercise. Patients are advised to contact their health care providers
when early symptoms of illness occur.
Activity 3
26. When did you first notice the rash?
What did the rash look like, when you first noticed it?
Where did it start?
Did it progress? If so, how?
Have you had this type of rash before? If so, how does it compare to this episode?
What makes the rash worse?
Is there anything that seems to make it better?
Are you having any other symptoms; for example, fever, coughing, congestion?
Have you used home remedies or over-the-counter medications to treat the rash? If so, what were they and did
they help?
Have you recently used any new lotions, soaps, or other personal care products?
Have you worn new clothes or brought any new textiles or furniture into the house?
Have you eaten any new foods?
Is anyone in the same household having the same kind of rash?
Do you have any pets? Do they go indoors and outdoors?
What do you do for work?
Are you exposed to chemicals or pollutants at work? If so, what are they?
Have you recently taken any trips, especially outside the United States?
CHAPTER 16—CARE OF THE PATIENT WITH HIV/AIDS
Short Answer
1. See Box 16-4 Common Opportunistic Diseases Associated With HIV
2. See Box 16-7 Barriers to Adherence
3. See Box 16-9 Conducting a Risk Assessment
Multiple Choice
4.
Answer 1, 2, 3, 4: HIV is transmitted from human to human through infected blood, semen, rectal secretions,
cervicovaginal secretions, and breast milk. HIV has been found in other body fluids such as saliva, urine, tears, and
feces, but there is no evidence that these substances are vehicles of transmission, unless the fluids contain visible
blood.
5.
Answer 1, 2, 4, 5, 6: HIV is not transmitted by casual contacts, such as hugging, dry kissing, shaking hands, or
sharing food and utensils. HIV is not transmitted by domestic animals or insects; through coughing or sneezing; or
through sharing objects such as pencils, computer keyboards, or telephones. HIV can be transmitted via
breastfeeding.
6.
Answer 1: Sexual intercourse is the most common mode of transmission and men who have sex with men and
engage in receptive anal intercourse comprise most of the HIV transmission in the US. The other groups of people
also need patient education information, but the prevention strategies need to be tailored to each group.
7.
Answer 2: Receptive anal intercourse is considered the most risky. The primary or late stages of the disease are
periods of the highest viral load and this also increases the risk. However, patients should be educated that
transmission can occur at any time and any transfer of semen or genital secretions offers potential risk.
8.
Answer 1, 2, 3, 4, 6: Drug users who inject could reduce risk by not sharing needles, but the lifestyle factors and
addiction to substances often result in sharing needles and other risky behaviors. Ease of access to safe sterile
equipment would reduce risk of HIV.
9.
10.
11.
12.
13.
14.
15.
16.
17.
18.
19.
20.
21.
22.
23.
24.
25.
Answer 1: While all of these incidents should be reported, the deep puncture wound with a hollow-bore needle full
of blood creates the greatest exposure.
Answer 3: Unfortunately, the antiviral prophylaxis can cause hepatitis, which may lead to liver failure and a
subsequent need for liver transplant.
Answer 2: In developed countries, antiretroviral therapy, formula feeding, and cesarean section have decreased the
numbers from 25% (without interventions) to approximately 1%.
Answer 3: For a CD4+ lymphocyte level of 200 cells/mm3 or less, opportunistic infections begin to emerge because
the body can no longer mount an adequate defense.
Answer 3: Typical progressors develop signs and symptoms several years after seroconverting. Long-term
nonprogressors may not develop signs and symptoms even 30 years after seroconverting. Rapid progressors move
from being infected with HIV to an AIDS diagnosis within 3 years.
Answer 4: In most people with HIV infection, the acute retroviral illness is mild and may be mistaken for a cold or
other minor viral infection. The viral load during the period of seroconversion is extremely high and the infected
person could transmit HIV to a partner without knowing it.
Answer 4: Although currently somewhat theoretical, a low viral set point appears to be associated with longer
survival times.
Answer 3: Alternative and complementary therapies can provide hope and relief from symptoms. The health care
team should be open to hearing about the patient’s interests and advise according to how they could fit in the
treatment plan.
Answer 1: If the phlebotomist is following Standard Precautions, there is no need to intervene. The nurse makes this
decision based on knowledge of Standard Precautions. (Note to the student: In the early days of HIV, the other
options were being used because there was fear and uncertainty surrounding HIV/AIDS.)
Answer 1, 2, 3, 4: For the patient with HIV, medications, infection, damage, and malabsorption contribute to
diarrhea. Hygiene and diet could be factors if the patient is noncompliant with basic health promotion instructions.
Answer 3: The HIV-associated cognitive motor complex will first produce mild memory deficits, similar to early
dementia. Physical impairments such as poor balance and coordination usually follow the cognitive impairments
and safety becomes the priority. The home health nurse has the expertise to identify safety issues (e.g., poor lighting,
throw rugs, cluttered walkways) that the family may not identify as problematic. Level of consciousness is usually
not affected. Numbness or tingling in hands or feet or pain in feet with walking are associated with peripheral
neuropathy.
Answer 1: Adolescents frequently believe in their invulnerability. Denial of risk would be typical.
Answer 2: Mutual masturbation would be the safest because there is no exchange of body fluids on the surface of
mucous membranes. Vaginal sex with consistent condom use is considered reasonably safe. Mutual monogamy is
only safe if both partners are mutually exclusive. Serial monogamy is considered risky, especially depending on
types of sexual activities/behaviors.
Answer 3: One of the most common infections in individuals with early symptomatic disease is oral candidiasis
(thrush), a fungal infection that is rare in healthy adults. Vaginal candidiasis (yeast infection) is a common
presenting condition for HIV-positive women.
Answer 2: With oral thrush, the mucous membranes of the mouth are tender and painful. Soft, moist foods are
recommended. Spicy or acidic foods will increase discomfort.
Answer 4: Oral hygiene includes, soft toothbrushes; nonabrasive toothpaste; and mouth rinses with sodium
bicarbonate, saline, or lemon and hydrogen peroxide before and after meals and at bedtime. Lesions should be
washed with a separate washcloth. Tub baths are avoided if rash is present. Nurse should assess for dependent
edema. See Box 16-6 for additional information.
Answer 3: The goal of palliative care is to relieve suffering through management of pain and symptoms. The nurse
is likely to assess the other issues and intervene as needed.
Critical Thinking Activities
Activity 1
26. a. The nursing student should be counseled about treatment options. The discussion should include recommended
medications, testing, testing intervals, home care, and follow-up.
b. The risk of exposure is highest if there is a deep injury involving a hollow-bore needle, that is filled with known
HIV-positive blood. If the infected patient is critically ill at the time of exposure, this also increases the risk.
c. Higher success will occur with rapid onset of preventive drug therapy. An exposed individual should start
therapy within 36 hours, but recommendations are to begin antiretroviral therapy as soon as possible.
d. Pros include: minimized chance of development of resistant virus, reduced HIV transmission risk, and
improved quality of life. Cons include: unpleasant side effects. liver damage, expense, and complexity of drug
regimen.
e. Living with family members will not put them at risk for HIV infection. Hugging, handholding, and sleeping
with family members will be safe. Unprotected sexual contact with her partner should be avoided.
Activity 2
27. The staff recognized that the worker had risks and attempted to offer her HIV testing. Only in rare circumstances,
such as the inability to give consent, can HIV antibody testing be completed without the patient’s informed consent.
Many ethical and legal issues surround HIV antibody testing; knowledge of applicable state laws is essential. In
many states, charges of assault and battery can be brought against health care workers who perform HIV testing
against a patient’s will. If the patient had agreed to HIV testing and been found positive, the clinic staff would have
faced another dilemma because of the worker’s occupation and the potential exposure to others. From the patient’s
point of view, she “tried to get her customers to use condoms” and she may have considered this the limit of her
liability towards infecting others. From the staff’s point of view, prostitution is illegal and her behavior did increase
risk for self and others; however, traditionally health care professionals do not refuse to treat prostitutes or notify
the police. Unfortunately, there is no method to inform her customers, unless she agrees to disclose their names.
Activity 3
28. HIV is now considered a chronic disease and Standard Precautions are the norm. As a contemporary nurse, you
may not feel any different about caring for an HIV/AIDS patient than you would about caring for any other patient.
However, health care workers have contracted HIV by work exposure, so all workers should be mindful of the risk.
Compared to the early days of HIV, there is more information, more treatment options, and less stigma
(although it still exists). Health care workers and patients are likely to feel more empowered by safety measures
such as needleless systems and heightened awareness of handling sharps. There are protocols for exposure that
guide workers in the event of an accidental needlestick or other types of exposure. In addition, there have been no
new confirmed reports of work-related exposure to health care workers since 2008.
CHAPTER 17—CARE OF THE PATIENT WITH CANCER
Matching
1.
g
2.
a
3.
f
4.
c
5.
b
6.
e
7.
d
Short Answer
8.
social, psychological, physical, and spiritual
9.
Common concerns of the patient include: (1) fear of recurrence, (2) chronic or acute pain, (3) sexual problems, (4)
fatigue, (5) guilt for delaying screening or treatment, (6) behavior that may have increased the risk for cancer, (7)
changes in physical appearance, (8) depression, (9) sleep problems, (10) change in role performance, and (11) being a
financial burden on loved ones
10. prostate; lung; colorectal
11. breast; lung; colorectal
12. See Box 17-1 Eight Warning Signs of Cancer
Figure Labeling
13. See Figure 17-3.
Clinical Application of Math
14. a. Answer 30 minutes
150 minutes ÷ 5 = 30 minutes
b. Answer 25 minutes
150 minutes ÷ 6 = 25 minutes
c. Answer 25 minutes
75 minutes ÷ 3 = 25 minutes
d. Answer 11 minutes
75 minutes ÷ 7 = 10.71 rounded to 11
15. Answer 7 pounds
140 pounds × 0.05 = 7 pounds
16. Answer 2 kg
123-118.5=4.5 pounds
4.5÷2.2=2.04 rounded to 2
Table Activity
17.
Male
Female
Erythrocytes
(RBCs)
4.7-6.1
million/mm3
4.2-5.4
million/mm3
Hemoglobin
14-18 g/dL
12-16 g/dL
Hematocrit
42-52%
37-47%
Multiple Choice
18. Answer 1: According to the American Cancer Society, smoking is the most preventable cause of death from lung
cancer. More cases of cancer occur among older adults. Many other cancers are associated with smoking.
19. Answer 4: Overall, African Americans have a higher incidence of cancer and death rate is higher among all minority
groups (except for Asian Americans).
20. Answer 2: All of these food trays contain some foods that reduce cancer risk, but option 2 is the best. For additional
information see Health Promotion Foods to Reduce Cancer Risk.
21. Answer 4: Women who have either of the genes BRCA1 and BRCA2 have as much as an 80% risk of having breast
cancer during their lifetime. These genes also increase risk for developing pancreatic, prostate (men), and male
breast cancer.
22. Answer 2: When an individual or family has characteristics suggestive of a hereditary cancer (such as cancers that
occur in several generations of a family, multiple primary cancers in one person, or rare cancers associated with
birth defects), a cancer risk assessment is performed. Genetic counseling is an essential component of the genetic
evaluation. The other people should be advised to follow up with their health care provider for evaluation and
diagnostic testing, but currently show less need for genetic counseling.
23. Answer 3: Some clinical manifestations of testicular cancer include a lump in or enlargement of either testicle, a
heavy feeling in the scrotum, dull aching in either the groin or the lower abdomen, fluid collection in the scrotum,
and tenderness or enlargement of the breasts.
24. Answer 2: Current or former smokers (quit within past 15 years) ages 55-74 in good health with at least a 30 packyear history are the target group for low dose helical CT. For additional information see Screening Guidelines for
the Early Detection of Cancer in Average-risk Asymptomatic People.
25.
26.
27.
28.
29.
30.
31.
32.
33.
34.
35.
36.
37.
38.
39.
40.
41.
42.
43.
44.
45.
46.
Answer 1, 2, 3, 4: Beginning at puberty, males should check the scrotum for enlargement, thickening, and the
presence of a lump in the testicles. This should be done monthly, after a warm bath or shower. In addition to selfexamination, men older than 50, need to discuss the prostate-specific antigen test with their health care provider and
undergo a rectal examination once a year.
Answer 4: Fruit and vegetable consumption is currently low in the United States. Fruits and vegetables are
particularly important in preventing gastrointestinal cancers, but also contain nutrients that decrease overall risk.
Answer 3: The nurse could suggest trying strawberries, peppers, tomatoes, or cantaloupe. Fresh food sources are
better than supplements. The patient might accept juice, but compliance is less likely since she dislikes citrus fruits.
Carrots and cauliflower are good anticancer vegetables, but offer less vitamin C.
Answer 4: The best time to perform breast self-examination (BSE) is 2-3 days after the end of the menses. The first
day of every month would be recommended to postmenopausal women. Women should not wait to see obvious
symptoms. The purpose of BSE is to detect subtle changes before obvious symptoms occur.
Answer 2: African Americans have a higher risk for prostate cancer and should be advised that age 40 is the time to
start prostate antigen testing.
Answer 4: Stage IV indicates metastasis.
Answer 1: T0; N0; M0 indicates no evidence of primary tumor, no regional lymph node metastasis, no (known)
distant metastasis. See Box 17-2 for additional information.
Answer 1: The radioisotope will concentrate in the tumor areas. Isotope that is not picked up by the bone can be
flushed out by the kidneys.
Answer 3: The history of hip fracture should be investigated prior to the MRI. If the patient has some type of metal
prosthesis in the hip, this would be a contraindication for MRI.
Answer 2: Alkaline phosphatase is elevated if there is liver disease or metastasis to the bone or liver. Serum
calcitonin is elevated in cancer of the thyroid. Normally, production of carcinoembryonic antigen (CEA) stops before
birth, but it may begin again if a neoplasm develops. CA-125 is a tumor marker for ovarian cancer.
Answer 1: Eating red meat, turnips, melons, aspirin, or vitamin C for 4 days before the test may cause a falsepositive result.
Answer 1: The nurse conveys respect but tries to remain available to help the patient. The nurse avoids offering
platitudes. The nurse could call the health care provider, but the patient is currently using the provider as a focus for
his anger. If the nurse is skilled at therapeutic communication, it is likely that the patient will be more comfortable
expressing his feelings to the nurse.
Answer 2: No lotion, cream, ointments, or powder should be applied over the markings. The markings must not be
washed off. If the skin should get wet, it should be patted dry.
Answer 2: The nurse must carefully plan the nursing care to limit the time spent in close contact with the patient.
The nurse can protect self by standing back, limiting time, and being very organized.
Answer 1: The patient is on bedrest and the UAP should only help with hygiene from the waist up. Time spent
should be limited. The patient should not be turned from side to side.
Answer 3: Catheterization should be avoided because it is a way to introduce infection. The nurse would check to
see if a midstream specimen would be adequate. The other interventions are correct.
Answer 3: Patients with chemo brain should be encouraged to use daily planners, participate in physical activity
and mental puzzles and should be encouraged to establish routines and avoid multi-tasking.
Answer 3: The patient’s mouth will be sore and irritated with open lesions. Frequent, gentle mouth care with a soft
brush or sponge and rinsing with normal saline will help. Cool fluids and bland foods are likely to feel more
soothing.
Answer 2: Epoetin alfa (Epogen) is used to treat anemia, which is reflected by the red cell count.
Answer 4: Platelets help the blood to clot; therefore, spontaneous bleeding will occur at a count of less than
20,000/mm3.
Answer 1: The hair loss is not permanent. It will grow back, but it may be a different color or texture.
Answer 1, 3, 4: The symptom should resolve when treatment ends. In the meantime, encourage the patient to
experiment with different spices: lemon juice, onion, mint, basil, and fruit juice marinades may improve the taste of
47.
48.
49.
50.
certain meats and fish. Ham and bits of bacon may improve the taste of vegetables. Calories are important, but good
nutrition is necessary for healing.
Answer 1, 2, 3, 6: People at different ages have different coping skills. If significant others are supportive and
symptoms are minimal, it is easier for the patient to cope. Ability to express feelings also helps the patient to cope.
Socioeconomic status and gender have less impact.
Answer 1: Ondansetron (Zofran) is an antiemetic, so the nurse will try to eliminate noxious odors.
Answer 2: Early clinical manifestations include nausea, vomiting, anorexia, diarrhea, muscle weakness, and
cramping. Later signs and symptoms may include tetany, paresthesias, seizures, anuria, and cardiac arrest.
Answer 3: For cancer patients, fixed-dose round-the-clock analgesia provides a constant blood level of the pain
medication. Bolus doses can be given for breakthrough pain, but fixed doses should continue, and the nurse should
report a pattern of continuous breakthrough to the health care provider for reevaluation of dose. Patient-controlled
analgesia and as needed medication may also be used.
Critical Thinking Activities
Activity 1
51.
a. Although the American Cancer Society recommends testing begin at age 50, the presence of a family history of
colon cancer may indicate the need to begin testing sooner. When an individual or family has characteristics
suggestive of a hereditary cancer (such as cancers that occur in several generations of a family, multiple primary
cancers in one person, or rare cancers associated with birth defects), a cancer risk assessment is performed. The
assessment begins with a comprehensive family history, including information on first-, second-, and thirddegree relatives. Medical records, death certificates, and autopsy reports may be obtained to help confirm the
cancer diagnoses previously identified in the family history.
b. The patient should be encouraged to add activity of at least 30 minutes per day into his routine. Dietary intake
should be evaluated. Fruits, vegetables, and whole grains should be encouraged, and fatty foods should be
avoided.
Activity 2
52. See Box 17-3 Instructions for Nursing Interventions for Patients Treated With Unsealed Internal Radiation for
Thyroid Cancer